Linear Algebra
Linear Algebra
Linear Algebra
c
David A. SANTOS
Preface v
1 Preliminaries 1
1.1 Sets and Notation . . . . . . . . . . . . . . . . . . . . . . . . . . 1
1.2 Partitions and Equivalence Relations . . . . . . . . . . . . . . 5
1.3 Binary Operations . . . . . . . . . . . . . . . . . . . . . . . . . . 9
1.4 Zn . . . . . . . . . . . . . . . . . . . . . . . . . . . . . . . . . . . 13
1.5 Fields . . . . . . . . . . . . . . . . . . . . . . . . . . . . . . . . . 18
1.6 Functions . . . . . . . . . . . . . . . . . . . . . . . . . . . . . . . 20
3 Linear Equations 79
3.1 Definitions . . . . . . . . . . . . . . . . . . . . . . . . . . . . . . 79
3.2 Existence of Solutions . . . . . . . . . . . . . . . . . . . . . . . . 84
3.3 Examples of Linear Systems . . . . . . . . . . . . . . . . . . . . 86
4 R2 , R3 and Rn 93
4.1 Points and Bi-points in R2 . . . . . . . . . . . . . . . . . . . . . 93
4.2 Vectors in R2 . . . . . . . . . . . . . . . . . . . . . . . . . . . . . 96
4.3 Dot Product in R2 . . . . . . . . . . . . . . . . . . . . . . . . . . 105
4.4 Lines on the Plane . . . . . . . . . . . . . . . . . . . . . . . . . 113
4.5 Vectors in R3 . . . . . . . . . . . . . . . . . . . . . . . . . . . . . 120
iii
iv CONTENTS
7 Determinants 183
7.1 Permutations . . . . . . . . . . . . . . . . . . . . . . . . . . . . . 183
7.2 Cycle Notation . . . . . . . . . . . . . . . . . . . . . . . . . . . 187
7.3 Determinants . . . . . . . . . . . . . . . . . . . . . . . . . . . . 194
7.4 Laplace Expansion . . . . . . . . . . . . . . . . . . . . . . . . . 205
7.5 Determinants and Linear Systems . . . . . . . . . . . . . . . . 213
These notes started during the Spring of 2002, when John MAJEWICZ
and I each taught a section of Linear Algebra. I would like to thank him
for numerous suggestions on the written notes.
v
Things done:
Now, can anyone, from anywhere in the world, help me with this?
Things to do:
4 Example Prove that between any two rational numbers there is al-
ways a rational number.
1
2 Chapter 1
a+c c
da + dc < cb + cd = d(a + c) < c(b + d) = < ,
b+d d
a+c
whence the rational number lies between a
b and d.
c
b+d
! We can also argue that the average of two distinct numbers lies between the
r1 + r2
numbers and so if r1 < r2 are rational numbers, then lies between them.
2
7 Definition (Quantifiers) The symbol is read for all (the universal quan-
tifier), and the symbol is read there exists (the existential quantifier).
We have
(x A, P(x)) ( A, P(x)) (1.1)
N Z Q R C.
! A = B (A B) (B A).
A B = {x : (x A) (x B)}.
A B = {x : (x A) (x B)}.
A \ B = {x : (x A) (x 6 B)}.
A B A B A B
(A B) C = (A C) (B C).
Solution: We have,
x (A B) C x (A B) x C
(x A x B) x C
(x A x C) (x B x C)
(x A C) (x B C)
x (A C) (B C),
A1 A2 An = {(a1 , a2 , . . . , an ) : ak Ak },
that is, the set of all ordered n-tuples whose elements belong to the
given sets.
! In the particular case when all the Ak are equal to a set A, we write
A1 A2 An = An .
If a A and b A we write (a, b) A2 .
t 0 = |x| t t x t. (1.4)
a R = a2 = |a|. (1.5)
|a| a |a|
to
|b| b |b|
we obtain
(|a| + |b|) a + b (|a| + |b|),
whence the theorem follows by 1.4.
Partitions and Equivalence Relations 5
25 Example Let
2Z = {. . . , 6, 4, 2, 0, 2, 4, 6, . . .} = 0
2Z + 1 = {. . . , 5, 3, 1, 1, 3, 5, . . .} = 1
26 Example Let
3Z = {. . . 9, , 6, 3, 0, 3, 6, 9, . . .} = 0
3Z + 1 = {. . . , 8, 5, 2, 1, 4, 7, . . .} = 1
6 Chapter 1
3Z + 2 = {. . . , 7, 4, 1, 2, 5, 8, . . .} = 2
! Notice that 0 and 1 do not mean the same in examples 25 and 26. Whenever
we make use of this notation, the integral divisor must be made explicit.
27 Example Observe
which means that the real numbers can be partitioned into the rational
and irrational numbers.
31 Example Let L be the set of all lines on the plane and write l1 l2 if
l1 ||l2 (the line l1 is parallel to the line l2 ). Then is an equivalence relation
on L.
Partitions and Equivalence Relations 7
a x x a
= ay = bx = xb = ya = .
b y y b
a b a2 + b2 > 2.
ab a2 + b2
b2 + a2
b a,
[a] = {x S : x a}.
Proof We prove that if (a, b) S2 , and [a][b] 6= then [a] = [b]. Suppose
that x [a] [b]. Now x [a] = x a = a x, by symmetry. Similarly,
x [b] = x b. By transitivity
(a x) (x b) = a b.
S= [a],
aS
and [a] [b] = if a b. This proves the first half of the theorem.
Conversely, let
S= S , S S = if 6= ,
transitive.
SS T
: .
(a, b) 7 (a, b)
We usually use the infix notation a b rather than the prefix notation
(a, b). If S = T then we say that the binary operation is internal or closed
and if S 6= T then we say that it is external. If
ab=ba
a (b c) = (a b) c,
a (b c) = (a b) c = a b c,
without ambiguity.
! We usually omit the sign and use juxtaposition to indicate the operation .
Thus we write ab instead of a b.
a b = 1 + ab = 1 + ba = ba,
! When we desire to drop the sign and indicate the binary operation by juxta-
position, we simply speak of the algebra S.
10 Chapter 1
47 Example Both hZ, +i and hQ, i are algebras. Here + is the standard
addition of real numbers and is the standard multiplication.
x (x y) = y, (1.7)
(y x) x = y. (1.8)
Shew that is commutative, but not necessarily associative.
Solution: By (1.8)
x y = ((x y) x) x.
By (1.8) again
By (1.7)
((x y) ((x y) y)) x = (y) x = y x,
which is what we wanted to prove.
0 (0 1) = 0 (0 1) = 0 (1) = 0 (1) = 1,
and
(0 0) 1 = (0 0) 1 = (0) 1 = 0 1 = 1,
evincing that the operation is not associative.
r = lr = l,
ax = ay = x = y.
54 Definition Let hS, i and hS, >i be algebras. We say that > is left-
distributive with respect to if
We say that > is distributive with respect to if it is both left and right
distributive with respect to .
55 Problem Let
S = {x Z : (a, b) Z2 , x = a3 + b3 + c3 3abc}.
x>y = x a y.
a b = a + b ab,
(x S)(x x = x),
Zn 13
and
1.4 Zn
Proof In the proof of this theorem, we use the following property of the
integers, called the well-ordering principle: any non-empty set of non-
negative integers has a smallest element.
S = {a bn : b Z a bn}.
10 9 8 7 6
5 4 3 2 1
0 1 2 3 4
5 6 7 8 9
.. .. .. .. ..
. . . . .
The arrangement above shews that any integer comes in one of 5 flavours:
those leaving remainder 0 upon division by 5, those leaving remainder 1
upon division by 5, etc. We let
0 = nZ, 1 = nZ + 1, 2 = nZ + 2, . . . , n 1 = nZ + n 1.
Zn = {0, 1, . . . , n 1}.
Our interest is now to define some sort of addition and some sort of
multiplication in Zn .
Zn 15
Now
a = a 0 = (q, q 0 ) Z2 , r N, a = qn + r, a 0 = q 0 n + r, 0 r < n,
0
b = b = (q1 , q10 ) Z2 , r1 N, b = q1 n + r1 , b 0 = q10 n + r1 , 0 r1 < n.
Hence
a + b = (q + q1 )n + r + r1 , a 0 + b 0 = (q 0 + q10 )n + r + r1 ,
a + b = a + b = a 0 + b 0 = a 0 + b 0.
Similarly
65 Example Let
Z6 = {0, 1, 2, 3, 4, 5}
be the residue classes modulo 6. Construct the natural addition + table
for Z6 . Also, construct the natural multiplication table for Z6 .
Solution: The required tables are given in tables 1.1 and 1.2.
+ 0 1 2 3 4 5 0 1 2 3 4 5
0 0 1 2 3 4 5 0 0 0 0 0 0 0
1 1 2 3 4 5 0 1 0 1 2 3 4 5
2 2 3 4 5 0 1 2 0 2 4 0 2 4
3 3 4 5 0 1 2 3 0 3 0 3 0 3
4 4 5 0 1 2 3 4 0 4 2 0 4 2
5 5 0 1 2 3 4 5 0 5 4 3 2 1
Table 1.1: Addition table for Z6 . Table 1.2: Multiplication table for Z6 .
67 Example Let
Z7 = {0, 1, 2, 3, 4, 5, 6}
be the residue classes modulo 7. Construct the natural addition + table
for Z7 . Also, construct the natural multiplication table for Z7
Solution: The required tables are given in tables 1.3 and 1.4.
+ 0 1 2 3 4 5 6 0 1 2 3 4 5 6
0 0 1 2 3 4 5 6 0 0 0 0 0 0 0 0
1 1 2 3 4 5 6 0 1 0 1 2 3 4 5 6
2 2 3 4 5 6 0 1 2 0 2 4 6 1 3 5
3 3 4 5 6 0 1 2 3 0 3 6 2 5 1 4
4 4 5 6 0 1 2 3 4 0 4 1 5 2 6 3
5 5 6 0 1 2 3 4 5 0 5 3 1 6 4 2
6 6 0 1 2 3 4 5 6 0 6 5 4 3 2 1
Table 1.3: Addition table for Z7 . Table 1.4: Multiplication table for Z7 .
Zn 17
5x = 3
in Z11 .
45x = 27,
that is,
x = 5.
gcd(a, b) = ax + by.
5x2 = 3
in Z11 .
1.5 Fields
74 Definition Let F be a set having at least two elements 0F and 1F
(0F 6= 1F ) together with two operations (multiplication, which we usually
represent via juxtaposition) and + (addition). A field hF, , +i is a triplet
satisfying the following axioms (a, b, c) F3 :
F1 Addition and multiplication are associative:
a + b = b + a, ab = ba (1.10)
a(b + c) = ab + ac (1.11)
0F + a = a + 0F = a (1.12)
1F a = a1F = a (1.13)
a1 F, aa1 = a1 a = 1F (1.15)
Fields 19
76 Example hQ, , +i, hR, , +i, and hC, , +i are all fields. The multiplicative
identity in each case is 1 and the additive identity is 0.
77 Example Let
Q( 2) = {a + 2b : (a, b) Q2 }
and define addition on this set as
(a + 2b) + (c + 2d) = (a + c) + 2(b + d),
and multiplication as
(a + 2b)(c + 2d) = (ac + 2bd) + 2(ad + bc).
Then hQ + 2Q, , +i is a field. Observe
0F = 0, 1F = 1, that the ad-
ditive
inverse of a + 2b is a 2b, and the multiplicative inverse of
a + 2b, (a, b) 6= (0, 0) is
1 1 a 2b a 2b
(a + 2b) = = 2 = 2 .
a + 2b a 2b2 a 2b2 a2 2b2
Here a2 2b2 6= 0 since 2 is irrational.
(a)1 = (a1 ).
1.6 Functions
85 Definition By a function or a mapping from one set to another, we
mean a rule or mechanism that assigns to every input element of the
first set a unique output element of the second set. We shall call the
set of inputs the domain of the function, the set of possible outputs the
Functions 21
target set of the function, and the set of actual outputs the image of the
function.
D T
f: .
x 7 f(x)
Here f is the name of the function, D is its domain, T is its target set,
x is the name of a typical input and f(x) is the output or image of x
under f. We call the assignment x 7 f(x) the assignment rule of the
function. Sometimes x is also called the independent variable. The set
f(D) = {f(a)|a D} is called the image of f. Observe that f(D) T.
1 2 1 4
2 8 2 2
3 4 3
X Y
86 Definition A function f : is said to be injective or one-
x 7 f(x)
to-one if (a, b) X2 , we have
a 6= b = f(a) 6= f(b).
This is equivalent to saying that
f(a) = f(b) = a = b.
R \ {1} R \ {1}
t: x+1
x 7
x1
22 Chapter 1
is an injection.
a+1 b+1
t(a) = t(b) = =
a1 b1
= (a + 1)(b 1) = (b + 1)(a 1)
= ab a + b 1 = ab b + a 1
= 2a = 2b
= a = b
1 4 1 4
2 2 2 2
3
8
! A function is surjective if its image coincides with its target set. It is easy to see
that a graphical criterion for a function to be surjective is that every horizontal line
passing through a point of the target set (a subset of the y-axis) of the function must
also meet the curve.
R R
91 Example Prove that t : is a surjection.
x 7 x3
Functions 23
Solution: Since the graph of t is that of a cubic polynomial with only one
zero, every horizontal line passing through a point in R will eventually
meet the graph of g, whence t is surjective. To prove this analytically,
proceed as follows. We must prove that ( b R) (a) such that t(a) = b.
We choose a so that a = b1/3 . Then
is an injection.
.. ..
. .
! As a shortcut, we often use the notation A = [aij ] to denote the matrix A with
entries aij . Notice that when we refer to the matrix we put parenthesesas in [aij ],
and when we refer to a specific entry we do not use the surrounding parenthesesas
in aij .
96 Example
0
A=
1 1
1 2 3
25
26 Chapter 2
is a 2 3 matrix and
2 1
B= 1 2
0 3
is a 3 2 matrix.
Solution: This is
1 2
11 12 22 12 32 12 42
0 1 2 3
2 4 = 1 2 .
A=
2
12 22 22 22 32 22 2
0 1
3
4 2 1
2
12 32 22 32 32 32
2
1 0
42 12 42 22 42 32 42 42 3 2 1 0
99 Definition The m n zero matrix 0mn Mmn (F) is the matrix with
0F s everywhere,
0 0F 0F 0F
0 0
F
F 0F 0F F
= 0 0 .
0mn
. 0F 0F
..
.. .
F F
.. ..
. .
0F 0F 0F 0F
F 1F 0F F
= 0 0 .
In
. 0F 1F
..
.. .
F F
.. ..
. .
0F 0F 0F 1F
1 1
1 1
102 Example For A = 1 1
and B = 2 1 we have
0 2 0 1
1 3
A + 2B = 3 3 .
0 0.
A+B=B+A (2.2)
A + (B + C) = (A + B) + C (2.3)
A + 0mn (2.4)
28 Chapter 2
M6 Distributive law
(A + B) = A + B (2.6)
M7 Distributive law
( + )A = A + B (2.7)
M8
1F A = A (2.8)
M9
(A) = ()A (2.9)
104 Problem Write out explicitly the 3 3 matrix A = [aij ] where aij = ij .
105 Problem Write out explicitly the 3 3 matrix A = [aij ] where aij = ij.
110 Problem A person goes along the rows of a movie theater and asks
the tallest person of each row to stand up. Then he selects the shortest
of these people, who we will call the shortest giant. Another person
goes along the rows and asks the shortest person to stand up and from
these he selects the tallest, which we will call the tallest midget. Who is
taller, the tallest midget or the shortest giant?
111 Problem [Putnam Exam, 1959] Choose five elements from the matrix
11 17 25 19 16
24 3
10 13 15
,
12 18
23 5 14 2
22
4 1 8
6 20 7 21 9
no two coming from the same row or column, so that the minimum of
these five elements is as large as possible.
. ..
.. ..
..
ai2 ain . . cij
..
i1
b . .
.. ..
. . n1 bnj bnp
am1 am2 amn cm1 cmp
30 Chapter 2
! Observe that we use juxtaposition rather than a special symbol to denote matrix
multiplication. This will simplify notation.In order to obtain the ij-th entry of the matrix
AB we multiply elementwise the i-th row of A by the j-th column of B. Observe that
AB is a m p matrix.
and N = 5 6be matrices over R. Then
113 Example Let M =
1 2
3 4 7 8
1 2 5 6= 1 5 + 2 7
MN =
1 6 + 2 8 19 22
= ,
3 4 7 8 35+47 36+48 43 50
and
5 6 1 2= 5 1 + 6 3
NM =
52+64
= 23 34.
7 8 3 4 71+83 72+84 31 46
2
1 1 1 0 0
1 0
1 0
1 0
1
1 1 = ,
1
2
0
0
1 1 1 1 1 2 0 0
over R. Observe then that the product of two non-zero matrices may be
the zero matrix.
1 0 2
.
4 1
=
0
3 3 1
116 Theorem If (A, B, C) Mmn (F) Mnr (F) Mrs (F) we have
(AB)C = A(BC),
Proof To shew this we only need to consider the ij-th entry of each side,
appeal to the associativity of the underlying field F and verify that both
sides are indeed equal to
n X
X r
aik bkk 0 ck 0 j .
k=1 k 0 =1
! By virtue of associativity, a square matrix commutes with its powers, that is, if
A Mn (F), and (r, s) N2 , then (Ar )(As ) = (As )(Ar ) = Ar+s .
Solution: The assertion is trivial for n = 1. Assume its truth for n 1, that is,
assume An1 = 3n2 A. Observe that
1 1 1 1 1 1 3 3 3
= 1 1 1 3
A2 1
1 1
= 3 3
= 3A.
1 1 1 1 1 1 3 3 3
Now
118 Theorem Let A Mn (F). Then there is a unique identity matrix. That
is, if E Mn (F) is such that AE = EA = A, then E = In .
In = EIn = E,
demonstrating uniqueness.
119 Example Let A = [aij ] Mn (R) be such that aij = 0 for i > j and
aij = 1 if i j. Find A2 .
n
X
bij = aik akj .
k=1
j
X
bij = aik akj = j i + 1.
k=i
Matrix Multiplication 33
.
A2
.. .. .. ..
. . . .
0 2
0 0 0 1
0 0 0 0 0 1
1
120 Problem Determine the product
1 2
1 1 1
.
1 1 0 1 1 2
1 0 0 a b c
121 Problem Let A = 1 0 B= b
1
, c a
. Find AB and BA.
1 1 1 b c a
122 Problem Let
2 3 4 1
1 1 1 1
1 4, 1 1
A= B=
2 3 1 1
4 3
1 1
1 2
1 1
3 4 1 2 1 1 1 1
x 4 0 1
over R.
124 Problem Prove or disprove! If (A, B) (Mn (F))2 are such that AB =
0n , then also BA = 0n .
34 Chapter 2
125 Problem Prove or disprove! For all matrices (A, B) (Mn (F))2 ,
(A + B)(A B) = A2 B2 .
1 1 = 1 n.
n
1
127 Problem Let M =
1
. Find M .
6
1 1
0 3. Find, with proof, A
128 Problem Let A = 2003
.
2 0
129 Problem Let (A, B, C) Mlm (F) Mmn (F) Mmn (F) and F.
Prove that
A(B + C) = AB + AC,
(A + B)C = AC + BC,
Prove that
1 n n(n+1)
=
2
An 0 1 n
.
0 0 1
n(n+1)
1 1 3 6
0 1 0
2
(n1)n
1 1
1 3 2
0 1 = 0
. (n2)(n1)
.
... ..
0 1 0 1
. .
2
.. ..
. .
.
..
.
..
.
0 0 0 1 0 0 0 1
134 Problem Let (A, B) (Mn (F))2 and k be a positive integer such that
Ak = 0n . If AB = B prove that B = 0n .
a b. Demonstrate that
135 Problem Let A =
c d
136 Problem Let A M2 (F) and let k Z, k > 2. Prove that Ak = 02 if and
only if A2 = 02 .
142 Example Let A Mn (R). Shew that A can be written as the sum of
two matrices whose trace is different from 0.
Solution: Write
A = (A In ) + In .
Now, tr (A In ) = tr (A) n and tr (In ) = n. Thus it suffices to take
tr (A)
6= , 6= 0. Since R has infinitely many elements, we can find such
n
an .
143 Example Let A, B be square matrices of the same size and over the
same field of characteristic 0. Is it possible that AB BA = In ? Prove or
disprove!
Trace and Transpose 37
145 Example If
a b c
,
M= d f
e
g h i
Then
ATT = A, (2.12)
(A + B)T = AT + BT , (2.13)
(AC)T = CT AT , (2.14)
Proof The first two assertions are obvious, and the fourth follows from the
third by using induction. To prove the third put AT = (ij ), ij = aji ,
CT = (ij ), ij = cji , AC = (uij ) and CT AT = (vij ). Then
n
X n
X n
X
uij = aik ckj = ki jk = jk ki = vji ,
k=1 k=1 k=1
148 Example Let A, B be square matrices of the same size, with A sym-
metric and B skew-symmetric. Prove that the matrix A2 BA2 is skew-
symmetric.
Solution: We have
(A + AT )T = AT + ATT = AT + A,
(A AT )T = AT ATT = (A AT ),
3 1 2
151 Problem Shew that there are no matrices (A, B, C, D) (Mn (R))4
such that
AC + DB = In ,
CA + BD = 0n .
152 Problem Let (A, B) (M2 (R))2 be symmetric matrices. Must their
product AB be symmetric? Prove or disprove!
Special Matrices 39
153 Problem Given square matrices (A, B) (M7 (R))2 such that tr A2 =
tr B2 = 1, and
(A B)2 = 3I7 ,
find tr (BA).
154 Problem Consider the matrix A =
a b M (R). Find necessary
2
c d
and sufficient conditions on a, b, c, d so that tr A = (tr (A)) .
2 2
and
(A I4 )2 = 3I4 ,
find tr (A).
158 Problem Let A be a square matrix. Prove that the matrix AAT is sym-
metric.
159 Problem Let A, B be square matrices of the same size, with A sym-
metric and B skew-symmetric. Prove that the matrix ABBA is symmetric.
is the set {0, 2, 7}. The counter diagonal of A is the set {5, 2, 9}.
165 Definition A square matrix is a diagonal matrix if every entry off its
main diagonal is 0F .
is a diagonal matrix.
0 0 4
is a scalar matrix.
that is, every element below the main diagonal is 0F . Similarly, A is lower
triangular if
((i, j) {1, 2, , n}2 ), (i < j, aij = 0F ),
that is, every element above the main diagonal is 0F .
Special Matrices 41
170 Example The matrix A M34 (R) shewn is upper triangular and B
M4 (R) is lower triangular.
1
1
0 0 0
1 a b c
0
A= 0
0 B=
a 0
2 3
0 2 3 0
0 0 0 1
1 1 t 1
172 Definition The set of matrices Eij Mmn (F), Eij = (ers ) such that
eij = 1F and ei 0 j 0 = 0F , (i 0 , j 0 ) 6= (i, j) is called the set of elementary matri-
ces. Observe that in fact ers = ir sj .
1 4 0 0 0
2 3
, 0 1.
A= 5 8 =
0
6 7 E23
0 0
9 10 11 12 0
0 0 0
Then
0 0 0 0
, 0 0 2
.
A= 9 12 = 6
E23 10 11
AE23 0 0
0 0 0 0 0 0 10
Proof Put (uv ) = Eij A. To obtain Eij A we multiply the rows of Eij by the
columns of A. Now
Xn Xn
uv = euk akv = ui kj akv = ui ajv .
k=1 k=1
Therefore, for u 6= i, uv = 0F , i.e., off of the i-th row the entries of Eij A
are 0F , and iv = jv , that is, the i-th row of Eij A is the j-th row of A. The
case for AEij is similarly argued.
175 Corollary Let (Eij , Ekl ) (Mn (F))2 , be square elementary matrices.
Then
Eij Ekl = jk Eil .
Solution: Assume (s, t) {1, 2, . . . , n}2 . Let M = (mij ) and Est Mn (F).
Since M commutes with Est we have
0. 0 ... 0 0 0 ... m1s ... 0
.. .. .. 0 .
0 ..
..
0
. ... .
. m2s .
..
m = E = ..
.. .. .. ..
.
... . . . .
mt2 ... mtn st M = MEst
0 0
t1
..
. ...
..
.
..
0 . m(n1)s
..
.
. ..
0 0 ... 0 0 0 .. mns . 0
For arbitrary s 6= t we have shown that mst = mts = 0, and that mss =
mtt . Thus the entries off the main diagonal are zero and the diagonal
entries are all equal to one another, whence M is a scalar matrix.
then
1 0 4 1 1 1 5 9 13
TA = 0 0 7 7
1
5 6
= 5 6
,
0 0 1 1 2 3 1 2 3
that is, pre-multiplication by T adds 4 times the third row of A to the first
row of A. Similarly,
1 1 1 1 0 4 1 1 5
AT = 5 7 0 27
6
0 1
= 5 6
,
1 2 3 0 0 1 1 2 7
Proof Simply observe that (In + Eij )A = A + Eij A and A(In + Eij ) =
A + AEij and apply Theorem 174.
then
4 0 0 1 1 1 4 4 4
SA = 0 0 7 7
1
5 6
= 5 6
,
0 0 1 1 2 3 1 2 3
183 Definition We write Iijn for the matrix which permutes the i-th row
with the j-th row of the identity matrix. We call Iij
n a transposition matrix.
Special Matrices 45
If
1 2 3 4
5 8 ,
A=
6 7
9 12
10 11
13 14 15 16
then
1 2 3 4
9 12 ,
A=
10 11
5
(23)
I4
8
6 7
13 14 15 16
and
1 3 2 4
5 8 .
=
7 6
9
(23)
12
AI4
11 10
13 15 14 16
then A = B.
Matrix Inversion 47
(X Mn (R)), ((XA)2 = 0n ).
Prove that A = 0n .
1
For
1 0
1 0
0
0
0 1
= 0 1 ,
0 1 0
x y
f g f g 0
(In ) 1 In = In = 1 In (In ) .
197 Corollary Let (A, B) (GLn (F))2 . Then AB is also invertible and
(AB)1 = B1 A1 .
B1 A1 (AB) = (AB)B1 A1 = In
and that since the inverse of a square matrix is unique, we must have
B1 A1 = (AB)1 .
The next few theorems will prove that elimination matrices are invertible
matrices.
Matrix Inversion 49
= In 2 ij Eij
= In ,
since i 6= j.
+(1 1F )Eii
+( 1F )(1 1F )Eii
= In + ( 1F )Eii
+(1 1F )Eii
+( 1F )(1 1F ))Eii
= In + ( 1F + 1 1F + 1F
1 1F ))Eii
= In ,
0 0 1 0 0 1 0 0 1
0
2 0
0 0
... ..
0 3 0
.
.. .. ..
. . .
0 0 0 0 n
is invertible and
1
0 0 0 0 1
0 0 0 0
0 0 0 0
1 1
1 0
2 0 0 2 0
0 0
= 0
. 0
... .. ..
0 3 0 0 1 0
. .. .
3
..
.
..
.
..
.
..
.
..
.
..
.
0 0 0 0 n 0 0 0 0 1
n
row with the j-th row, meaning that they return to the original position in
In . Observe in particular that Iij
n = (In ) , and so In (In ) = In .
ij T ij ij T
Matrix Inversion 51
1 0 0 1 0 0 1 0 0
0 1 1 0
0
0 0
= 0 1
.
0 1 0 0 1 0 0 0 1
Proof This follows from Corollary 197, and Theorems 200, 202, and 205.
1 0 0
A= 0 4
3
0 0 1
1 0 0 1
= 0 0 0 1
0 0 0
,
0 4 4 0
3
1
3
0 0 1 0 0 1 0 0 1
and so
52 Chapter 2
1 1 1
1 0 0 1 0 0
0 1 4
0 0
=
1
3
0 0 1 0 0 1
1 0 0
.
0
=
1 4
3 3
0 0 1
hence 1 1 1
1 0 0 1 1 0
0 1 0
0 1 1
=
0 0 1 0 0 1
1 1 0
.
0 1 1
=
0 0 1
In the next section we will give a general method that will permit us
Matrix Inversion 53
c d c a 0 1
211 Example If
1 1 1 1
1 1 ,
A=
1 1
1 1
1 1
1 1 1 1
then A is invertible, for an easy computation shews that
2
1 1 1 1
1 1 = 4I ,
=
1 1
A2
1 1
4
1 1
1 1 1 1
whence the inverse sought is
1 1 1 1
1/4 1/4 1/4 1/4
1 1 1 = 1/4 1/4 .
= A=
1 1 1/4 1/4
4 1
1
A1
4
1 1 1 1/4
1/4 1/4 1/4
1 1 1 1 1/4 1/4 1/4 1/4
1
(1 x)1 = = 1 + x + x2 + x3 + .
1x
(A + A2 + A3 + + Ak )
= In
and
= In .
is
3 0 0
= 0 0
A1 2
,
0 0 4
as
2 0 0 3
0 0 0 1
= 0 0 0
= 0 0 0 0
1
AA 3 2 1
0 0 4 0 0 4 0 0 1
Matrix Inversion 55
Proof If A were invertible, the (i, i)-th entry of the product Pof its inverse
n
with A would be 1F . But if the i-th row of A is all 0F s, then k=1 aik bki =
0F , so the (i, i) entry of any matrix product with A is 0F , and never 1F .
1 1 1
216 Problem The inverse of the matrix A = 1 2
is the matrix A
1
1 =
1 2 3
a 1 1
1 b 1 . Determine a and b.
1 1 0
219 Problem Let S GLn (F), (A, B) (Mn (F))2 , and k a positive integer.
Prove that if B = SAS1 then Bk = SAk S1 .
220 Problem Let A Mn (F) and let k be a positive integer. Prove that A
is invertible if and only if Ak is invertible.
56 Chapter 2
221 Problem Let S GLn (C), A Mn (C) with Ak = 0n for some positive
integer k. Prove that both In SAS1 and In S1 AS are invertible and
find their inverses.
222 Problem Let A and B be square matrices of the same size such that
both A B and A + B are invertible. Put C = (A B)1 + (A + B)1 . Prove
that
ACA ACB + BCA BCB = 2A.
a b b b
b b
a b
225 Problem Let A = b b M (F), n > 1, (a, b) F .
... ..
2
b a n
.
.. ..
. .
b b b a
Determine when A is invertible and find this inverse when it exists.
226 Problem Let (A, B) (Mn (F))2 be matrices such that A + B = AB.
Demonstrate that A In is invertible and find this inverse.
227 Problem Let S GLn (F) and A Mn (F). Prove that tr (A) = tr SAS1 .
228 Problem Let A Mn (R) be a skew-symmetric matrix. Prove that In +
A is invertible. Furthermore, if B = (In A)(In + A)1 , prove that B1 = BT .
! If (A, A0 ) (Mm (F))2 , (B, B0 ) (Mmn (F))2 , (C, C0 ) (Mnm (F))2 , (D, D0 )
(Mm (F))2 , and
S=
A
C
B
D
, T= A0
C0
B0
D0
,
ST = AA0 + BC0
CA0 + DC0
AB0 + BD0
CB0 + DD0
.
Proof Assume first that A, and B are invertible. Direct calculation yields
A C A
1
A 1
CB 1
=
AA 1
AA 1
CB 1
+ CB 1
0rm 0rm B1 0 BB1
B rm
=
I m 0mr
0rm Ir
= Im+r .
58 Chapter 2
Assume now that L is invertible, L1 =
E H, with E M
m (F) and
J K
K Mr (F), but that, say, B is singular. Then
I m 0mr
= LL1
0rm Ir
=
A C E H
0
rm B J K
=
AE + CJ AH + BK,
BJ BK
Proof We prove the result for row equivalence. The result for column
equivalence, and equivalence are analogously proved.
called the Hermite normal form of A. Thus there exist P GLm (F), Q
GLn (F) such that Dm,n,r = PAQ. The integer r 0 is called the rank of
the matrix A which we denote by rank (A).
.. .. ..
F . . .
Here the asterisks represent unknown entries. Observe that the bs form
a (m 1) (n 1) matrix.
Observe that this results in a matrix of the form
1
0
F
F 1F
= 0 .
P2 AQ2
0F c33 c3n
(2.18)
0
F
.. .. ..
F . . .
0F 0F cm3 cmn
GJ-6 Add the appropriate multiple of column 1 to column 2, that is, ap-
ply a transvection, in order to make the entry in the (1, 2) position
0F .
This now gives a matrix of the form
1 0F
0
F
F 1F
= 0 .
P3 AQ3
0F c33 c3n
(2.19)
0
F
.. .. ..
F . . .
0F 0F cm3 cmn
R= S ,
11 R12
S1 21 S22 S23
R21 R22
S31 S32 S33
with (R11 , S11 )2 (Mr (F))2 , S22 M(sr)(sr) (F). We have
RDm,n,r
R
=
11 R12
Ir 0(mr)r
= R 11 0(mr)r
,
R21 R22 0(mr)r 0r(mr) R21 0r(mr)
Rank of a Matrix 61
and
I 0r(sr) 0r(ns) S S12 S13
0
r 11
S
1
Dm,n,s S = (sr)r Isr 0(sr)(ns) 21 S22 S23
0 0(ms)(sr) 0(ms)(ns)
(ms)r S 31 S32 S33
S S12 S13
S
11
=
21 S22 S23
.
0(ms)r 0(ms)(sr) 0(ms)(ns)
R =
S11 S12 S13
0(mr)r
,
11
S21 S22 S23
R21 0r(mr)
0(ms)r 0(ms)(sr) 0(ms)(ns)
S 1
S 21 0(sr)(sr) 0(sr)(ns)
.
S31 S32 S33
The matrix
0 (sr)(sr) 0(sr)(ns)
S32 S33
! Albeit the rank of a matrix is unique, the matrices P and Q appearing in Theorem
234 are not necessarily unique. For example, the matrix
1 0
0 1
0 0
62 Chapter 2
0 1 y
0 0 1
is invertible, and an easy computation shews that
1 0 x 1 0 1 0
0 1 y 0 1
1
0
0
1
= 0 1 ,
0 0 1 0 0 0 0
regardless of the values of x and y.
0
Solution: We first transpose the first and third columns by effecting
3
0 3 0
0 1
2
0 1 0
= 0
2
.
0 1 0 1 0
1 0 0
We now subtract twice the second row from the first, by effecting
1 2 3
2 0 3
= 0 0
.
0 1 0 1 0 0 1 0
Rank of a Matrix 63
We conclude that
0
1
1/3 0 1 2 0 3 0
0 1
2
0 1 0
= 0
0
,
0 1 0 1 0 1 0 1 0
1 0 0
and
0 0 1
Q= 0 0
1
.
1 0 0
Exchanging the i-th row with the j-th row, which we denote by Ri
Rj , and the s-th column by the t-th column by Cs Ct .
1 0
0 0.
A=
1 1
1 2
Solution: First observe that rank (A) min(4, 2) = 2, so the rank can be
either 1 or 2 (why not 0?). Form the augmented matrix
1 0 0 0 0 0
0 0
1 0 0 0
1 0
.
0
0 1 0 0
0 0 1 0 0
1 0
1 0 0 1
1 2 0 0 0 1
1 0 0 0 0 0
0 0
1 0 0 0
1 0
.
0
0 1 0 0
0 0 1 0 0
0 0
1 1 0 1
0 2 1 0 0 1
Rank of a Matrix 65
Perform R6 2R5 R6
1 0 0 0 0 0
0 0
1 0 0 0
1 0
.
0
0 1 0 0
0 0 1 0 0
0 0
1 1 0 1
0 0 1 0 2 1
Perform R4 R5
1 0 0 0 0 0
0 0
1 0 0 0
1 0
.
0
0 1 0 0
1 1 0 1 0
0 0
0 0 1 0
0 0 1 0 2 1
Finally, perform R3 R3
1 0 0 0 0 0
0 0
1 0 0 0
1 0
.
0
0 1 0 0
1 1 0 1 0
0 0
0 0 1 0
0 0 1 0 2 1
We conclude that
1 0 0 0
1 0
1 0
1 0 0 0 1 0= 0 1 .
0 1
0 0
0 1 0 0 1
1 0 1
1 0 2 1 1 2 0 0
66 Chapter 2
Proof We prove that rank (A) rank (AB). The proof that rank (B)
rank (AB) is similar and left to the reader. Put r = rank (A) , s = rank (AB).
There exist matrices P GLm (F), Q GLn (F), S GLm (F), T GLp (F)
such that
PAQ = Dm,n,r , SABT = Dm,p,s .
Now
and
Dm,p,s
V=
Ir 0r(nr)
V 11 V12
Mmp (F).
0(mr)r 0(mr)(nr) V21 V22
If s > r then (i) U11 would have at least one row of 0F s meaning that U11
is non-invertible by Lemma 215. (ii) U21 = 0(ms)s . Thus from (i) and (ii)
and from Lemma 231, U is not invertible, which is a contradiction.
Rank of a Matrix 67
and so rank (B) = rank (AB). A similar argument works when B is invert-
ible.
241 Example Study the various possibilities for the rank of the matrix
1 1 1
A= b + c a+b
c+a
.
bc ca ab
Solution: Performing R2 (b + c)R1 R2 and R3 bcR1 R3 , we find
1 1 1
0
ab ac
.
0 0 (b c)(a c)
Performing C2 C1 C2 and C3 C1 C3 , we find
1 0 0
0
ab ac
.
0 0 (b c)(a c)
We now examine the various ways of getting rows consisting only of 0s.
If a = b = c, the last two rows are 0-rows and so rank (A) = 1. If exactly
two of a, b, c are equal, the last row is a 0-row, but the middle one is not,
and so rank (A) = 2 in this case. If none of a, b, c are equal, then the rank
is clearly 3.
a + 1 a+2 a+3 a+4 a+5
a + 2 a+6 M (R).
243 Problem Find the rank of
a+3 a+4 a+5
a + 3 a+7
5
245 Problem Study the various possibilities for the rank of the matrix
1 a 1 b
a 1
1 b
1 b 1 a
b 1 a 1
when (a, b) R2 .
1 1 0 1
m 1 as a function of m C.
246 Problem Find the rank of
1 1
1 0
m 1
1 1 m 2
a 2
ab ab b2
ab ab .
247 Problem Determine the rank of the matrix
a2 b2
ab ab
b2 a2
b2 ab ab a2
1 1 1 1
a b .
248 Problem Determine the rank of the matrix
b a
c d
c d
ac bc ad bd
Rank and Invertibility 69
249 Problem Let A M32 (R), B M2 (R), and C M23 (R) be such that
1 1 2
ABC = 2 1
x
. Find x.
1 2 1
250 Problem Let A, B be matrices of the same size. Prove that rank (A + B)
rank (A) + rank (B).
251 Problem Let B be the matrix obtained by adjoining a row (or col-
umn) to a matrix A. Prove that either rank (B) = rank (A) or rank (B) =
rank (A) + 1.
252 Problem Let A Mn (R). Prove that rank (A) = rank AAT . Find a
counterexample in the case A Mn (C).
Conversely, assume that rank (A) = n. Then rank AT = n by Corol-
lary 235, and so by Theorem 234 there exist P GLm (F), Q GLn (F), such
that
PAQ =
In
,
QT AT PT = In 0n(mn) .
0(mn)n
This gives
QT AT PT PAQ = In = AT PT PA = (QT )1 Q1
= ((QT )1 Q1 )1 AT PT PA = In ,
70 Chapter 2
255 Corollary If A Mmn (F) possesses a left inverse L and a right in-
verse R then m = n and L = R.
! If A Mn (R) is non-invertible, then the left hand side in the procedure above will
not reduce to In .
6 0 1
.
B= 3 0
2
1 0 1
Rank and Invertibility 71
Solution: We have
6 0 1 1 0 0 1 0 1 0 0 1
3 0 3 0
R1 R3
2 0 0 1
2 0 0 1
1 0 1 0 0 1
6 0 1 1 0 0
1 0 1 0 0 1
0 4
R3 6R1 R3
R2 3R1 R2 2 4 0 1
0 0 2 1 0 1
5 0 0 1 0 6
0 2
R2 2R3 R2
5R1 +R3 R1 2 0 5 1
0 0 2 1 0 1
1 0 0 3 0 4
.
0 1
3R1 R1 ; 4R3 R3
4R2 R2 1 0 6 4
0 0 1 4 0 4
We conclude that
1
6 0 1 3 0 4
3 0 = 1
2
6 4
.
1 0 1 4 0 4
257 Example Use Gau-Jordan reduction to find the inverse of the ma-
0 1 1
trix A = 4 . Also, find A
3 4
2001
.
3 3 4
72 Chapter 2
0 1 1 1 0 0
0 1 1 1 0 0
4 0 1 0 0 1 1
R2 R3 R2
3 4 0 1
0
3 3 4 0 0 1
3 3 4 0 0 1
0 1 1 1 0 0
1 0 0 1 1
R3 3R2 R3
0
0 3 4 0 3 4
0 1 1 1 0 0
1 0 0 0 1 1
R3 +3R1 R3
0 0 1 3 3 4
0 1 0 4 3 4
1 0 0 0 1 1
R1 +R3 R1
0 0 1 3 3 4
1 0 0 0 1 1
.
0 1 0 4 3 4
R1 R2
0 0 1 3 3 4
0 1 1
= A.
= 4 4
1
A 3
3 3 4
0 0 0 1
.
.. .. .. .. .. ..
. . . . . .
0 0 0 1 0 0 0 1
.
.. .. .. .. .. ..
. . . . . .
0 0 0 1 0 0 0 1
whence
1 1 0 0
0 0
1 1
= 0 0 ,
A1
. 0 1
..
.. .
.. ..
. .
0 0 0 1
that is, the inverse of A has 1s on the diagonal and 1s on the super-
diagonal.
74 Chapter 2
259 Theorem Let A Mn (F) be a triangular matrix such that a11 a22 ann 6=
0F . Then A is invertible.
and so rank (AB) = 2. This entails that rank (AB)2 = 2. Now, since BA is
a 2 2 matrix, rank (BA) 2. Also
2 = rank (AB)2 = rank (ABAB) rank (ABA) rank (BA) ,
Rank and Invertibility 75
and we must conclude that rank (BA) = 2. This means that BA is invert-
ible and so
= BA 9I2 = 02
261 Problem Find the inverse of the matrix
1 2 3
2 1
3
M (Z ).
3 7
3 1 2
263 Problem Find all the values of the parameter a for which the matrix
B given below is not invertible.
1 a+2 2
B= 0 1
a
2 1 a
0 0 c
76 Chapter 2
0 2 n
1
1 1 ... 1 1 1 ... 1
1 1 2 n
0 1 ...
1 1 ... 1
1 1
n1
1
1
... .. ...
1 0 ... = 1 2n ... 1
.
.. .. .. .. ..
. . ... . . ... .
1 1 1 ... 0 1 1 1 ... 2n
..
.
..
. ...
..
.
1 1 1 ... 1+a
Rank and Invertibility 77
has inverse
1 n a 1 1 ... 1
1 n a
1 1 ... 1
a(n + a)
1
1
...
1 1 n a ... 1
..
.
..
. ...
..
.
1 1 1 ... 1na
.. .. .. .. ..
. . . . .
3 5 7 9 1
has inverse
2 n 2
2 + n2 2 2 2
2
2 n2 2 + n2 2 2
1
2 .
...
2 2 n2 2 + n2 2
2n3
..
.
..
.
..
.
..
.
..
.
2 + n2 2 2 2 2 n2
1 + a2 1 ... 1
1
...
1 1 + a3 ... 1
..
.
..
. ...
..
.
1 1 1 ... 1 + an
78 Chapter 2
has inverse
1a s
a
1 1 1 1
...
1 1 a s
2 a1 a2 a1 a3 a1 an
1
a a a
2 1 1
1
...
1 ,
2 a2 a3 a2 an
2 1 2
s a a n
1 1 a3 s 1
a ..a a3 a2 a23
...
.
3 1 3 1
.. .. ..
1 1
.
1
. ...
...
.
1a s n
an a1 an a2 an a3 a2n
where s = 1 + 1
a1 + 1
a2 + + an .
1
271 Problem Let A M5 (R). Shew that if rank A2 < 5, then rank (A) < 5.
272 Problem Let A M3,2 (R) and B M2,3 (R) be matrices such that
0 1 1
AB = 1 1
0
. Prove that BA = I . 2
1 1 2
Chapter 3
Linear Equations
3.1 Definitions
We can write a system of m linear equations in n variables over a field F
..
.
in matrix form as
a 11 a12 a1n
x y
1 1
AX = Y, (3.2)
79
80 Chapter 3
variables X and will simply write the augmented matrix of the system as
a 11 a12 a1n y1
a y .
[A|Y] = . ..
a22 a2n
..
21 2
(3.3)
.
.. .. ..
. . .
am1 am2 amn ym
are in row-echelon form, with the pivots circled, but the matrices
1 0 1 1
1 0 1 1
0 2, 0 0,
0 1
0 0
0 0 1 1
0 0 0 1
0 0 0 0 0 0 0 0
x
1 1 3
0
1 1
0
y = 1 .
2 1
z
0 0 1 1 4
w
z w = 4 = z = 4 + w = 4 + t,
1 1 1 1 5 1
2y + z = 1 = y = z = 2 t = t,
2 2 2 2 2 2
5 1 9 3
x + y + z + w = 3 = x = 3 y z w = 3 + + t 4 t t = t.
2 2 2 2
The solution is thus
y t
9 3
x t
2 2
= , t R.
5 1
z 4 + t
2 2
w t
Solution: We see that x, y are leading variables, while z, w are free pa-
rameters. We put z = s, w = t. Operating from the bottom up, we find
1 1 1 1
2y + z = 1 = y = z = s,
2 2 2 2
5 3
x + y + z + w = 3 = x = 3 y z w = s t.
2 2
Definitions 83
= , (s, t) R .
1 1
z s
2 2 2
w t
x + 2y + 2z = 0,
y + 2z = 1,
working in Z3 .
y = 1 2z = 1 + 1z,
and
x = 2y 2z = 1 + 2z.
Thus
x 1 + 2z
y = 1 + 1z ,
z Z3 .
z z
x 0
y = 2 ,
z 1
and
x 2
y = 0 .
z 2
x + y + z + w = 0,
2y + w = 2.
1x + 2y + 3z = 5;
2x + 3y + 1z = 6;
3x + 1y + 2z = 0.
284 Lemma Let A Mmn (F) be in row-echelon form, and let X Mn1 (F)
be a matrix of variables. The homogeneous system AX = 0m1 of m lin-
ear equations in n variables has (i) a unique solution if m = n, (ii) multiple
solutions if m < n.
Existence of Solutions 85
so there is only the unique solution X = 0n1 , called the trivial solution.
285 Theorem Let A Mmn (F), and let X Mn1 (F) be a matrix of vari-
ables. The homogeneous system AX = 0m1 of m linear equations in n
variables always has a non-trivial solution if m < n.
That is, the systems AX = 0m1 and BX = 0m1 have the same set of
solutions. But by Lemma 284 there is a non-trivial solution.
x
X = . .
..
2
xn
1 21
Cn+1 .. i i
. i=1
Now assume that r = rank (A) = rank ([A|Y]). This means that adding
an extra column to A does not change the rank, and hence, by a se-
quence column operations [A|Y] is equivalent to [A|0n1 ]. Observe that
none of these operations is a permutation of the columns, since the first
n columns of [A|Y] and [A|0n1 ] are the same. This means that Y can be
obtained from the columns Ci , 1 i n of A by means of transvections
and dilatations. But then
Xn
Y= xi Ci .
i=1
The solutions sought is thus
x 1
x
X = . .
..
2
xn
x + 2y + 3z + 4w = 8
x + 2y + 4z + 7w = 12
2x + 4y + 6z + 8w = 16
Examples of Linear Systems 87
Solutions: Form the expanded matrix of coefficients and apply row op-
erations to obtain
1 2 3 4 8
1 2 3 4 8
.
1 12 0 4
R3 2R1 R3
2 4 7 R2 R1 R2 0 1 3
2 4 6 8 16 0 0 0 0 0
The matrix is now in row-echelon form. The variables x and z are the
pivots, so w and y are free. Setting w = s, y = t we have
z = 4 3s,
x = 8 4w 3z 2y = 8 4s 3(4 3s) 2t = 4 + 5s 2t.
Hence the solution is given by
y t
x 4 + 5s 2t
= .
z 4 3s
w s
z t
2 0
2 0
1 0 1 2
6 0 1 1
1 0 1 2
0 1
R3 6R1 R3
R2 3R1 R2 2 4
0 0 2 3
Examples of Linear Systems 89
This gives
2z = 3 = z = 5,
2y = 1 4z = 2 = y = 1,
x = 2 z = 4.
The solution is thus
(x, y, z) = (4, 1, 5).
4
2 4 2
d 0
1 0 0 0 1 f 0
4
1 4 4
d 6
1 2 3 4 5 f 9
if any.
x + 2my + z = 4m;
2mx + y + z = 2;
x + y + 2mz = 2m2 ,
with real parameter m. You must determine, with proof, for which m this
system has (i) no solution, (ii) exactly one solution, and (iii) infinitely many
solutions.
90 Chapter 3
294 Problem Study the following system of linear equations with param-
eter a.
(2a 1)x + ay (a + 1)z = 1,
ax + y 2z = 1,
You must determine for which a there is: (i) no solution, (ii) a unique
solution, (iii) infinitely many solutions.
295 Problem Determine the values of the parameter m for which the
system
x + y + (1 m)z = m+2
(1 + m)x y + 2z = 0
2x my + 3z = m+2
is solvable.
296 Problem Determine the values of the parameter m for which the
system
x + y + z + t = 4a
x y z + t = 4b
x y + z + t = 4c
x y + z t = 4d
is solvable.
ay + bx = c;
cx + az = b;
bz + cy = a
x3 y2 z6 = 1
x4 y5 z12 = 2
x2 y2 z5 = 3.
x5 + x2 = yx1 ;
x1 + x3 = yx2 ;
x2 + x4 = yx3 ;
x3 + x5 = yx4 ;
x4 + x1 = yx5 ,
where y is a parameter.
92 Chapter 3
Chapter 4
R2, R3 and Rn
Given A =
a 1
R2 and B =
b 1
R2 we define their addition
a2 b2
as
a 1
b 1
a 1 + b1
(4.1)
A+B= + = .
a2 b2 a2 + b2
a 1
a 1
(4.2)
A = = .
a2 a2
! Throughout this chapter, unless otherwise noted, we will use the convention that
93
94 Chapter 4
a point A R2 will have its coordinates named after its letter, thus
A=
a1
a2
.
a 1
a
A=
2
O
x
0
[A, A] = O = .
0
! The bi-point [A, B] can be thus interpreted as an arrow starting at A and finishing,
with the arrow tip, at B. We say that A is the tail of the bi-point [A, B] and that B is its
head. Some authors use the terminology fixed vector instead of bi-point.
302 Definition Let A 6= B be points on the plane and let L be the line
passing through A and B. The direction of the bi-point [A, B] is the direc-
tion of the line L, that is, the angle 2 ; 2 that the line L makes with
the horizontal. See figure 4.2.
303 Definition Let A, B lie on line L, and let C, D lie on line L 0 . If L||L 0
then we say that [A, B] has the same direction as [C, D]. We say that the
bi-points [A, B] and [C, D] have the same sense if they have the same
Points and Bi-points in R2 95
direction and if both their heads lie on the same half-plane made by the
line joining their tails. They have opposite sense if they have the same
direction and if both their heads lie on alternative half-planes made by
the line joining their tails. See figures 4.3 and 4.4 .
B B B
D
A A
C
A
C
Figure 4.2: Direction of a Figure 4.3: Bi-points with Figure 4.4: Bi-points with
bi-point the same sense. opposite sense.
! A bi-point is completely determined by three things: (i) its norm, (ii) its direction,
and (iii) its sense.
and
[A, A] = O.
2. [A, B] has the sense of [A, B] if > 0 and sense opposite [A, B] if
< 0.
3. [A, B] has norm ||||[A, B]|| which is a contraction of [A, B] if 0 < || <
1 or a dilatation of [A, B] if || > 1.
2[A, B]
B [A, B]
A
C
1
[A, B]
2
4.2 Vectors in R2
307 Definition (Midpoint) Let A, B be points in R2 . We define the mid-
point of the bi-point [A, B] as
a1 +b1
A+B 2
= .
2 a2 +b2
2
Vectors in R2 97
308 Definition (Equipollence) Two bi-points [X, Y] and [A, B] are said to
be equipollent written [X, Y] [A, B] if the midpoints of the bi-points [X, B]
and [Y, A] coincide, that is,
X+B Y+A
[X, Y] [A, B] = .
2 2
See figure 4.7.
X ||
|| B
A
Figure 4.7: Equipollent bi-points.
309 Lemma Two bi-points [X, Y] and [A, B] are equipollent if and only if
y 1 x1
b 1 a1
= .
y2 x2 b2 a2
b1 +x1
2 2
[X, Y] [A, B] =
a2 +y2 b2 +x2
2 2
y 1 x1
b 1 a1
= ,
y2 x2 b2 a2
as desired.
! From Lemma 309, equipollent bi-points have the same norm, the same direction,
and the same sense.
Proof Write [X, Y] [A, B] if [X, Y] if equipollent to [A, B]. Now [X, Y] [X, Y]
since
y 1 x1
y1 x1
and so the relation is reflexive. Also
=
y2 x2 y2 x2
y 1 x1
b 1 a1
[X, Y] [A, B] =
y2 x2 b2 a2
b 1 a1
y 1 x1
=
b2 a2 y2 x2
y 1 x1
b 1 a1
[X, Y] [A, B] [A, B] [U, V] =
y2 x2 b2 a2
b 1 a1
v 1 u1
=
b2 a2 v2 au2
y 1 x1
v 1 u1
=
y2 x2 v2 u2
311 Definition (Vectors on the Plane) The equivalence class in which the
bi-point [X, Y] falls is called the vector (or free vector) from X to Y, and is
denoted by XY. Thus we write
y
[X, Y] XY =
1 x
.1
y2 x2
write
v
v = .
1
v2
!
For any point X on the plane, we have XX = 0, the zero vector. If [X, Y] v then
[Y, X] v.
p2
R2
p
we may form the vector OP = . We call OP the position vector of
1
p2
P and we use boldface lowercase letters to denote the equality OP = p.
313 Example The vector into which the bi-point with tail at A =
1
2
and head at B =
3 falls is
4
AB =
3 (1)
= 4.
42 2
1
3
A= ,B = ,
2 4
3
7
X= ,Y =
7 9
represent the same vector
3 (1)= 4= 7 3= XY.
AB =
42 2 97
100 Chapter 4
u2 v2
and a scalar R then their sum and scalar multiplication take the form
u v
u+ v = + ,
1 1 u
u = .
1
u2 v2 u2
2u
B u
u v
A
C
u+v
1
u
2
315 Example Diagonals are drawn in a rectangle ABCD. If AB = x and
AC = y, then BC = y x, CD = x, DA = x y, and BD = y 2x.
316 Definition (Parallel Vectors) Two vectors u and v are said to be par-
allel if there is a scalar such that u = v. If u is parallel to v we write u||v.
We denote by Rv = {v : R}, the set of all vectors parallel to v.
317 Definition If u =
u , then we define its norm as ||u|| = u
1
2
1 + u22 .
u2
The distance between two vectors u and v is dhu, vi = ||u v||.
318 Example Let a R, a > 0 and let v 6= 0. Find a vector with norm a
and parallel to v.
v
Solution: Observe that has norm 1 as
||v||
v ||v||
||v|| = ||v|| = 1.
v
Hence the vector a has norm a and it is in the direction of v. One
||v||
v
may also take a .
||v||
319 Example If M is the midpoint of the bi-point [X, Y] then XM = MY
from where XM = 21 XY. Moreover, if T is any point, by Chasles Rule
TX + TY = TM + MX + TM + MY
= 2TM XM + MY
= 2TM.
320 Example Let 4ABC be a triangle on the plane. Prove that the line
joining the midpoints of two sides of the triangle is parallel to the third
side and measures half its length.
102 Chapter 4
AC. Thus
BC = BA + AC
= AB + AC
= 2AMC + 2AMB
= 2MC A + 2AMB
= 2(MC A + AMB )
= 2MC MB ,
as we wanted to shew.
321 Example In 4ABC, let MC be the midpoint of side AB. Shew that
CMC =
2
1
CA + CB .
Solution: Since AMC = MC B, we have
CA + CB = CMC + MC A + CMC + MC B
= 2CMC AMC + MC B
= 2CMC ,
322 Theorem (Section Formula) Let APB be a straight line and and
be real numbers such that
||[A, P]||
= .
||[P, B]||
With a = OA, b = OB, and p = OP, then
b + a
p= . (4.4)
+
AP = AO + OP = a + p.
whence
[A, P] = [A, B] = AP = AB = p a = (b a).
+ + +
( + )(p a) = (b a) = ( + )p = b + a,
1 a.
and
1
324 Problem Find all scalars for which ||v|| = 1
2
1
, where v = .
1
325 Problem Given a pentagon ABCDE, find AB + BC + CD + DE + EA.
will be parallel?
327 Problem In 4ABC let the midpoints of [A, B] and [A, C] be MC and
MB , respectively. Put MC B = x, MB C = y, and CA = z. Express [A]
AB + BC + MC MB , [B] AMC + MC MB + MB C, [C] AC + MC A BMB in
terms of x, y, and z.
104 Chapter 4
328 Problem A circle is divided into three, four equal, or six equal parts
(figures 4.10 through 4.15). Find the sum of the vectors. Assume that
the divisions start or stop at the centre of the circle, as suggested in the
figures.
a a a
c d
b c c b
b
a a b c
c d c d a d
f e
b b
329 Problem Diagonals are drawn in a square (figures 4.16 through 4.18).
Find the vectorial sum a + b + c. Assume that the diagonals either start,
stop, or pass through the centre of the square, as suggested by the fig-
ures.
a b
a b b a
c c c
330 Problem Prove that the mid-points of the sides of a skew quadrilat-
eral form the vertices of a parallelogram.
Dot Product in R2 105
332 Problem Let A, B be two points on the plane. Construct two points I
and J such that
1
IA = 3IB, JA = JB,
3
and then demonstrate that for any arbitrary point M on the plane
MA + 3MB = 4MI
and
3MA + MB = 4MJ.
The following properties of the dot product are easy to deduce from the
definition.
DP1 Bilinearity
DP3 Commutativity
xy = yx (4.7)
DP4
x x 0 (4.8)
DP5
xx = 0 x = 0 (4.9)
DP6
||x|| = xx (4.10)
a2
a = a1 i + a2 j.
The vectors
1 0
i = , j = ,
0 1
337 Theorem
ab = ||a||||b|| cos (a, b).
Dot Product in R2 107
||b a||2 = ||a||2 + ||b||2 2||a||||b|| cos (a, b)
(b a)(b a) = ||a||2 + ||b||2 2||a||||b|| cos (a, b)
a b = ||a||||b|| cos (
a, b),
as we wanted to shew.
Putting (a, b) =
2 in Theorem 337 we obtain the following corollary.
338 Corollary Two vectors in R2 are perpendicular if and only if their dot
product is 0.
ba
a
339 Definition Two vectors are said to be orthogonal if they are perpen-
dicular. If a is orthogonal to b, we write a b.
|ab| ||a||||b||.
Proof
||a + b||2 = (a + b)(a + b)
= aa + 2ab + bb
= (||a|| + ||b||)2 ,
= aa + 2ab + bb
= aa + 0 + bb
= ||a||2 + ||b||2 ,
where (v, t) [0; ] is the convex angle between v and t read in the
positive sense.
Dot Product in R2 109
! Given two vectors t and vector v 6= 0, find bi-point representatives of them hav-
ing a common tail and join them together at their tails. The projection of t onto v is the
shadow of t in the direction of v. To obtain projtv we prolong v if necessary and drop
a perpendicular line to it from the head of t. The projection is the portion between
the common tails of the vectors and the point where this perpendicular meets t. See
figure 4.20.
projxa = cos (x, a)||x||
1
||a||
a=
xa
||a||
2
a.
av = ax aprojxa
x a
ax a ||a|| 2a
=
= ax xa
= 0,
a = sv + s 0 v 0 ,
a = sv + tw.
sv + tw = a = pv + qw.
a = sp + tz
= sp tprojqp + tq
= s t p + tq,
qp
||p||2
q p
establishing the result upon choosing l = s t ||p|| 2 and m = t.
1 1
350 Example Let p = , q = . Write p as the sum of two vectors,
1 2
one parallel to q and the other perpendicular to q.
2
3
q = q = .
3
5
||q|| 5 6
5
Dot Product in R2 111
We also compute
p projpq
1 = .
=
3
5
2
5
6
1 5 15
Observe that
= 6 6 = 0,
3
5
2
5
6
5125 25
5
and the desired decomposition is
B
H
A
351 Example Prove that the altitudes of a triangle 4ABC on the plane
are concurrent. This point is called the orthocentre of the triangle.
Solution: Put a = OA, b = OB, c = OC. First observe that for any x, we
have, upon expanding,
(x a)(b c) + (x b)(c a) + (x c)(a b) = 0. (4.11)
Let H be the point of intersection of the altitude from A and the altitude
from B. Then
0 = AHCB = (OH OA)(OB OC) = (OH a)(b c), (4.12)
and
0 = BHAC = (OH OB)(OC OA) = (OH b)(c a). (4.13)
Putting x = OH in (4.11) and subtracting from it (4.12) and (4.13), we
gather that
0 = (OH c)(a b) = CHAB,
which gives the result.
112 Chapter 4
a be perpendicular
352 Problem Determine the value of a so that
1a
1
to .
1
4 1 2
354 Problem Let p = , r = , s = . Write p as the sum of two
5 1 1
vectors, one parallel to r and the other parallel to s.
a + b = 0 = = = 0.
v R2 , va = vb,
then a = b.
uv=
1
4
||u + v||2 ||u v||2 .
Lines on the Plane 113
projax
proj = a,
a
with 0 1.
It is clear that the points A, B, and C are collinear if and only if AB is
parallel to AC. Thus we have the following definition.
y p2
x
If r = , then the equation of the line can be written in the form
y
r p = tv. (4.14)
The Cartesian equation of a line is an equation of the form ax + by = c,
where a2 + b2 6= 0. We write (AB) for the line passing through the points
A and B.
365 Theorem Let v 6= 0 and let n v. An alternative form for the equa-
tion of the line r p = tv is
(r p)n = 0.
114 Chapter 4
Moreover, the vector
a is perpendicular to the line with Cartesian
b
equation ax + by = c.
Proof The first part follows at once by observing that vn = 0 and taking
dot products to both sides of 4.14. For the second part observe that at
least one of a and b is 6= 0. First assume that a 6= 0. Then we can put y = t
and x = a
b
t + ac and the parametric equation of this line is
x = t ,
c
a
b
a
y 1
and we have
a= b a + b = 0.
b
a
1 a b
and we have
1 a= a a b = 0,
a b b
b
! The vector
a
a 2 +b 2
b
has norm 1 and is orthogonal to the line ax + by = c.
a 2 +b 2
4
the direction of v = is
5
x 2= 4.
y3 5
1
Put v = and w = . Since the lines are perpendicular we must
1
m1 m2
have vw = 0, which yields
0 = vw = 1(1) + m1 (m2 ) = m1 m2 = 1.
|(a b)n|
.
||n||
|ab1 + bb2 c|
.
a2 + b2
Proof Let R0 be the point on the line that is nearest to B. Then BR0 = r0 b
is orthogonal to the line, and the distance we seek is
(r b) n |(r b) n|
=
||projnr0 b ||
||n|| n = ||n|| .
0 0
as we wanted to shew.
0 b
y
Lines on the Plane 117
We use the result obtained above with a =
, n = a, and B =
c
a
0 b
b 1
. Then ||n|| =
a2 + b2 and
b2
b a
|(a b) n| =
b b = |c ab
c
a 1
1 bb2 | = |ab1 + bb2 c|,
2
371 Example Recall that the medians of 4ABC are lines joining the ver-
tices of 4ABC with the midpoints of the side opposite the vertex. Prove
that the medians of a triangle are concurrent, that is, that they pass
through a common point.
1 1 1
r0 (AB + BC) s0 (AB + AB + BC) = AB,
2 2 2
s0 r0 s0
(r0 + 1)AB = ( + )BC.
2 2 2
We must have
s0
r0 +
1 = 0,
2
r0 s0
+ = 0,
2 2
since otherwise the vectors AB and BC would be parallel, and the trian-
gle would be degenerate. Solving, we find s0 = r0 = 32 . Thus we have
2
OA + 3 AMA = OG, or AG = 32 AMA , and similarly, BG = 23 BMB .
2
From AG = 3 AMA ,
we deduce AG = 2GMA . Since MA is the mid-
point of [B, C], we have GB + GC = 2GMA = AG, which is equivalent
to
GA + GB + GC = 0.
As MC is the midpoint of [A, B] we have GA + GB = 2GMC . Thus
0 = GA + GB + GC = 2GMC + GC.
This means that GC = 2GMC , that is, that they are parallel, and so the
points G, C and MC all lie on the same line. This achieves the desired
result.
373 Problem Find the equation of the line passing through
1 and
1
2
in a direction perpendicular to .
1
Lines on the Plane 119
375 Problem Let ABCD be a trapezoid, with bases [A, B] and [C, D]. The
lines (AC) and (BD) meet at E and the lines (AD) and (BC) meet at F.
Prove that the line (EF) passes through the midpoints of [A, B] and [C, D]
by proving the following steps.
Let I be the midpoint of [A, B] and let J be the point of intersection
of the lines (FI) and (DC). Prove that J is the midpoint of [C, D].
Deduce that F, I, J are collinear.
Prove that E, I, J are collinear.
AE =
3
1
AB + AD
and prove that the points A, C, E are collinear.
378 Problem Put OA = a, OB = b, OC = c. Prove that A, B, C are
collinear if and only if there exist real numbers , , , not all zero, such
that
a + b + c = 0, + + = 0.
4.5 Vectors in R3
We now extend the notions studied for R2 to R3 . The rectangular coor-
dinate form of a vector in R3 is
a
a= a .
1
a3
In particular, if
1 0 0
i= 0, j = 1, k = 0
0 0 1
a
then we can write any vector a = a as a sum
1
a3
a = a1 i + a2 j + a3 k.
a b
Given a = a and b = b , their dot product is
1 1
2 2
a3 b3
ab = a1 b1 + a2 b2 + a3 b3 ,
and
||a|| = a 2
1 + a22 + a23 .
We also have
ij = jk = ki = 0,
and
||i|| = ||j|| = ||k|| = 1.
j j
i i
CP1 Anti-commutativity:
xy = (yx) (4.15)
CP2 Bilinearity:
CP4
xx = 0 (4.18)
ij = k, jk = i, ki = j (4.19)
122 Chapter 4
x y
382 Theorem Let x = x and y = y be vectors in R . Then
1 1
3
2 2
x3 y3
(x1 i + x2 j + x3 k)(y1 i + y2 j + y3 k) = x1 y2 ij + x1 y3 ik
+x2 y1 ji + x2 y3 jk
+x3 y1 ki + x3 y2 kj
= x1 y2 k x1 y3 j x2 y1 k
+x2 y3 i + x3 y1 j x3 y2 i,
Solution: We have
= k 2j 3i + 60
= 3i 2j + k.
Hence
1 0 3
0 1 = 2 .
3 2 1
Proof We will only check the first assertion, the second verification is
analogous.
= x1 x2 y3 x1 x3 y2 + x2 x3 y1 x2 x1 y3 + x3 x1 y2 x3 x2 y1
= 0,
Proof
= a1 (b3 c1 b1 c3 )k a1 (b1 c2 b2 c1 )j
a2 (b2 c3 b3 c2 )k + a2 (b1 c2 b2 c1 )i
= (a1 c1 + a2 c2 + a3 c3 )(b1 i + b2 j + b3 i)
+(a1 b1 a2 b2 a3 b3 )(c1 i + c2 j + c3 i)
= (ac)b (ab)c,
387 Theorem Let (x, y) [0; [ be the convex angle between two vectors
x and y. Then
||xy|| = ||x||||y|| sin (x, y).
Proof We have
x
y
x y
393 Problem Find the area of the triangle whose vertices are at P =
0 0 1
0 ,Q= 1 , and R = 0 .
1 0 0
397 Problem The vectors a, b are constant vectors. Solve the equation
a(xb) = b(xa).
398 Problem The vectors a, b, c are constant vectors. Solve the system
of equations
2x + ya = b, 3y + xa = c,
126 Chapter 4
399 Problem Prove that there do not exist three unit vectors in R3 such
2
that the angle between any two of them be > .
3
Proof This follows at once from Corollary 349, since the operations occur
on a plane, which can be identified with R2 .
z
non-collinear, AB and AC, which are coplanar, are non-parallel. Since
AR also lies on the plane, we have by Lemma 401, that there exist real
numbers p, q with
AR = pAB + qAC.
By Chasles Rule,
OR = OA + p(OB OA) + q(OC OA),
is the equation of a plane containing the three non-collinear points A,
B, and C. By letting r = OR, a = OA, etc., we deduce that
r a = pu + qv.
x a1 = pu1 + qv1 ,
y a2 = pu2 + qv2 ,
z a3 = pu3 + qv3 .
403 Example Find both the parametric equation and the Cartesian equa-
1 1
tion of the plane parallel to the vectors 1and 1and passing through
1 0
0
the point 1 .
(r a)n = 0.
128 Chapter 4
a
Moreover, the vector bis normal to the plane with Cartesian equation
c
ax + by + cz = d.
Proof The first part is clear, as un = 0 = vn. For the second part, recall
that at least one of a, b, c is non-zero. Let us assume a 6= 0. The argument
is similar if one of the other letters is non-zero and a = 0. In this case we
can see that
d b c
x= y z.
a a a
x d
b c
y
a a a
= s 1 + t 0
z 0 1
405 Example Find once again, by appealing to Theorem 404, the Carte-
1 1
sian equation of the plane parallel to the vectors 1and 1and pass-
1 0
0
ing through the point 1 .
1 1 1
Solution: The vector 1 1= 1 is normal to the plane. The plane
1 0 0
Planes and Lines in R3 129
z2 0
as obtained before.
|(a b)n|
.
||n||
Proof Let R0 be the point on the plane that is nearest to B. Then BR0 =
r0 b is orthogonal to the plane, and the distance we seek is
(r b) n |(r b) n|
=
||projnr0 b ||
||n|| n = ||n|| .
0 0
as we wanted to shew.
! Given three planes in space, they may (i) be parallel (which allows for some of
them to coincide), (ii) two may be parallel and the third intersect each of the other
two at a line, (iii) intersect at a line, (iv) intersect at a point.
r a = tv, t R.
408 Theorem Put OA = a, OB = b, and OC = c. Points (A, B, C) (R3 )3
are collinear if and only if
ab + bc + ca = 0.
130 Chapter 4
Proof If the points A, B, C are collinear, then AB is parallel to AC and by
Corollary 388, we must have
(c a)(b a) = 0.
Rearranging, gives
cb ca ab = 0.
Further rearranging completes the proof.
giving
||(a b)v||
||r0 b|| = ,
||v||
proving the theorem.
! Given two lines in space, one of the following three situations might arise: (i) the
lines intersect at a point, (ii) the lines are parallel, (iii) the lines are skew (one over the
other, without intersecting).
410 Problem Find the equation of the plane passing through the points
(a, 0, a), (a, 1, 0), and (0, 1, 2a) in R3 .
411 Problem Find the equation of plane containing the point (1, 1, 1)
and perpendicular to the line x = 1 + t, y = 2t, z = 1 t.
Planes and Lines in R3 131
412 Problem Find the equation of plane containing the point (1, 1, 1)
and containing the line x = 2y = 3z.
413 Problem Find the equation of the plane perpendicular to the line
ax = by = cz, abc 6= 0 and passing through the point (1, 1, 1) in R3 .
414 Problem Find the equation of the line perpendicular to the plane
ax + a2 y + a3 z = 0, a 6= 0 and passing through the point (0, 0, 1).
x y z = 1, x z = 1,
416 Problem Find the equation of the plane passing through the points
1 2 x 1 1
0 , 1 and parallel to the line y= 2+ t 0..
1 1 z 0 1
418 Problem Find the equation of the plane which is equidistant of the
3 1
points 2 and 1 .
1 1
2x + 4y + 3z 36,
2x + 3z 24.
4.7 Rn
As a generalisation of R2 and R3 we define Rn as the set of n-tuples
x1
x2
: x R .
.. i
.
xn
x y
x y = . .. = x y
..
2 2
.
1 1 + x2 y2 + + xn yn .
xn yn
|xy| ||x||||y||.
n
X n
X n
X
Proof Put a = x2k , b = xk yk , and c = y2k . Consider
k=1 k=1 k=1
n
X n
X n
X n
X
f(t) = (txk yk )2 = t2 x2k 2t xk yk + y2k = at2 + bt + c.
k=1 k=1 k=1 k=1
Rn 133
4 x y 4 x
n 2 n n
X X X
y .
2 2
k k k k
k=1 k=1 k=1
This gives
2 2
|xy|2 ||x|| ||y||
from where we deduce the result.
n 4 n n n 2
X X X X
k=1
ak bk ck
k=1
a4k
k=1
b4k
k=1
c2k
.
Pn
Solution: Using CBS on k=1 (ak bk )ck once we obtain
X a b X c .
n n 1/2 n 1/2
X
2 2 2
ak bk ck k k k
k=1 k=1 k=1
a b P c
P n P n 2 2
1/2 n 2
1/2
a b c
a P b P
k=1 k k k k=1 k k k=1 k
P n 4
1/4 n 4
1/4 n 1/2
k=1 k k=1 k k=1 c2k ,
422 Theorem (Triangle Inequality) Given (x, y) (Rn )2 the following in-
equality holds
||x + y|| ||x|| + ||y||.
Proof We have
= aa + 2ab + bb
= (||a|| + ||b||)2 ,
n 1/p
X
(4.20)
p
||x||p = |xk |
k=1
Clearly
||x||p 0 (4.21)
||x||p = 0 x = 0 (4.22)
1 1
423 Lemma (Youngs Inequality) Let p > 1 and put + = 1. Then for
p q
(a, b) ([0; +[)2 we have
ap aq
ab + .
p q
[0; +[ R
f: .
x 7 xk k(x 1)
bq/p
a
1+
1
p
ap
bq
!
1 .
Rearranging gives
ap b1+p/qp b1+p/q
ab b1+p/q +
p p
424 Theorem (Hlder Inequality) Given (x, y) (Rn )2 the following inequal-
ity holds
|xy| ||x||p ||y||q .
Adding, we deduce
Pn |xk | |yk | 1 Pn 1 Pn
k=1 p |xk |p + q |yk |q
||x||p ||y||p ||x||p p k=1 ||y||q q k=1
p q
||x||p ||y||q
= p + q
||x||p p ||y||q q
1 1
= +
p q
= 1.
This gives
n
X
|xk yk | ||x||p ||y||q .
k=1
X
The result follows by observing that
X
x y
n
|x y | ||x|| ||y|| .
n
k=1
k k
k=1
k k p q
425 Theorem (Minkowski Inequality) Let p ]1; +[. Given (x, y) (Rn )2
the following inequality holds
||x + y||p ||x||p + ||y||p .
|x | # #
k=1 k=1 k=1
= "P n
k=1 k "P
p 1/p n
k=1 |xk + yk |p
1/q
p/q
= ||x||p ||x + y||p
(4.25)
In the same manner we deduce
n
X p/q
|yk ||xk + yk |p1 ||y||p ||x + y||p . (4.26)
k=1
1k<jn
(ak bj aj bk )2
Pn
427 Problem Let ai Rn for 1 i n be unit vectors with i=1 ai = 0.
P n
Prove that 1i<jn ai aj = .
2
428 Problem Let ak > 0. Use the CBS Inequality to shew that
X a X a1 n .
n n
2 2
k 2
k=1 k=1 k
X a
n 2 n
n(n + 1)(2n + 1) X a2k
k .
6 k2
k=1 k=1
Chapter 5
Vector Spaces
a + b V, (5.1)
a V, (5.2)
VS3 Commutativity
a+b=b+a (5.3)
VS4 Associativity
(a + b) + c = a + (b + c) (5.4)
0V :a+0=a+0=a (5.5)
137
138 Chapter 5
VS9
1F a = a (5.9)
VS10
()a = (a) (5.10)
(a1 , a2 , . . . , an ) = (a1 , a2 , . . . , an ).
In particular, is a vector space with only four elements and
hZ22 , +, , Z2 i
we have seen the two-dimensional and tridimensional spaces hR2 , +, , Ri
and hR3 , +, , Ri.
433 Example If
F[x] = {a0 + a1 x + a2 x + + an xn : ai F, n N}
434 Example If
Fn [x] = {a0 + a1 x + a2 x + + ak xk : ai F, n N, k n}
435 Example Let k N and let Ck (R[a;b] ) denote the set of k-fold contin-
uously differentiable real-valued functions defined on the interval [a; b].
Then Ck (R[a;b] ) is a vector space under addition of functions and multi-
plication of a function by a scalar.
Vector Spaces 139
436 Example Let p ]1; +[. Consider the set of sequences {an }
n=0 , an
C,
X
lp = {an }n=0 : |an |p < + .
n=0
{an }
n=0 + {bn }n=0 = {(an + bn )}n=0 ,
{an }
n=0 = {an }n=0 , C.
All the axioms of a vector space follow trivially from the fact that we are
adding complex numbers, except that we must prove that P in lp there
is closure under
P addition and scalar multiplication. Since n=0 |an |p <
n=0 |an | < + closure under scalar multiplication follows
p
+ =
easily. To prove closure under addition, observe that if z C then |z| R+
and so by the Minkowski Inequality Theorem 425 we have
P N
n=0 |an + bn |p 1/p
P N
n=0 |an |p
1/p
+ PN
n=0 |bn |p 1/p
P P (5.11)
1/p 1/p
( n=0 |an |p ) + ( n=0 |bn |p ) .
This in turn implies that the series on the left in (5.11) converges, and so
we may take the limit as N + obtaining
1/p 1/p 1/p
X X X
(5.12)
p p p
|an + bn | |an | + |bn | .
n=0 n=0 n=0
Now (5.12) implies that the sum of two sequences in lp is also in lp , which
demonstrates closure under addition.
F, 0 = 0.
140 Chapter 5
Proof We have
0 = (0 + 0) = 0 + 0.
Hence
0 0 = 0,
or
0 = 0,
proving the theorem.
v V, 0F v = 0.
Proof We have
0F v = (0F + 0F )v = 0F v + 0F v.
Therefore
0F v 0F v = 0F v,
or
0 = 0F v,
proving the theorem.
v = 0 = = 0F v = 0.
v = 0 = 1 v = 1 0.
1 v = 0.
Proof We have
0F v = ( + ())v = v + ()v,
whence
(v) + 0F v = ()v,
that is
(v) = ()v.
Similarly,
0 = (v v) = v + (v),
whence
(v) + 0 = (v),
that is
(v) = (v),
x2 y2 x2 + y2 x2 0
a vector space?
444 Problem Let V = R+ =]0; +[, the positive real numbers and F = R,
the real numbers. Demonstrate that V is a vector space over F if vector
addition is defined as a b = ab, (a, b) (R+ )2 and scalar multiplication
is defined as a = a , (, a) (R, R+ ).
445 Problem Let C denote the complex numbers and R denote the real
numbers. Is C a vector space over R under ordinary addition and multi-
plication? Is R a vector space over C?
Proof Let F and (a, b) (X Y)2 . Then clearly (a, b) X and (a, b)
Y. Since X is a vector subspace, a + b X and since Y is a vector
subspace, a + b Y. Thus
a + b X Y
! We we will soon see that the only vector subspaces of hR2 , +, , Ri are the set
containing the zero-vector, any line through the origin, and R2 itself. The only vector
subspaces of hR3 , +, , Ri are the set containing the zero-vector, any line through the
origin, any plane containing the origin and R3 itself.
is a vector subspace of R4 .
a + 2b
a
is a vector subspace of R5 .
X = {x Rn : ax = 0}
is a subspace of Rn .
144 Chapter 5
X = {x R3 : ax = 0}
is a subspace of R3 .
459 Problem Prove that the set X Mn (F) of upper triangular matrices is
a subspace of Mn (F).
462 Problem Prove that the following subsets are not subspaces of the
given vector space. Here you must say which of the axioms for a vector
space fail.
a
2 2
: a, b R, a + b = 1 R3
b
0
a
2 3
b : a, b R , ab = 0 R
0
a b: (a, b) R , a + b = 0 M
2 2
22 (R)
0 0
465 Problem Let V a vector space over a field F. If F is infinite, show that
V is not the set-theoretic union of a finite number of proper subspaces.
for
a b= a 1 0+ b 0 1+ c 0 0+ d 0 0.
c d 0 0 0 0 1 0 0 1
! A family of vectors is linearly independent if and only if the only linear combina-
tion of them giving the zero-vector is the trivial linear combination.
472 Example
1
4 7
2
3
,
5 , 8
6 9
(a + b)u + (a b)v = 0.
474 Theorem Let A Mmn (F). Then the columns of A are linearly inde-
pendent if and only the only solution to the system AX = 0m is the trivial
solution.
477 Problem Prove that the set
1
1 1 1
1
1 1 1
, , ,
1 1 1 0
1
1 1 1
1
2
and shew that X = can
is a linearly independent set of vectors in R4
1
1
be written as a linear combination of these vectors.
148 Chapter 5
478 Problem Let (a, b) (R3 )2 and assume that ab = 0 and that a and
b are linearly independent. Prove that a, b, ab are linearly indepen-
dent.
480 Problem Let (u, v) (Rn )2 . Prove that |uv| = ||u||||v|| if and only if u
and v are linearly dependent.
{v1 + v2 , v2 + v3 , v3 + v4 , v4 + v1 }
484 Problem Is the family {1, 2} linearly independent over Q?
485 Problem Is the family {1, 2} linearly independent over R?
Spanning Sets 149
2. Express
1 2
+
1 2 12 2
as a linear combination of {1, 2, 3}.
{w, u1 , u2 , . . . , uk , . . . , } V
also spans V.
c
a
b = ai + bj + ck.
c
spans R3 .
c 0 0 1
1 0, 0 1, 0 0, 0 0 is a spanning set for
the set of matrices
0 0 0 0 1 0 0 1
M2 (R)
span (u1 , u2 , . . . , uk , . . . , ) .
span (u1 , u2 , . . . , uk , . . . , ) V
is a vector subspace of V.
{u1 , u2 , . . . , uk , . . . , }.
{u1 , u2 , . . . , uk , . . . , }
498 Example If A M (R), A span
1 0, 0 0, 0 1 then A
1 0
2
0 0 0 1
has the form
1 0+ b 0 0+ c 0 1= a c,
a
0 0 0 1 1 0 c b
i.e., this family spans the set of all symmetric 2 2 matrices over R.
499 Theorem Let V be a vector space over a field F and let (v, w) V 2 ,
F \ {0F }. Then
span (v, w) = span (v, w) .
av + bw = av + (b1 )(w),
500 Theorem Let V be a vector space over a field F and let (v, w) V 2 ,
F. Then
span (v, w) = span (w, v + w) .
av + bw = a(v + w) + (b a)w.
501 Problem Let R3 [x] denote the set of polynomials with degree at most
3 and real coefficients. Prove that the set
spans R3 [x].
1 1 0
502 Problem Shew that 1 6 span 0 , 1
.
1 1 1
Bases 153
1 0, 0 0, 0 1 ?
503 Problem What is span
0 0 0 1 1 0
prove that
span (a, b) = span (c, d) .
5.5 Bases
506 Definition A family {u1 , u2 , . . . , uk , . . .} V is said to be a basis of V if
(i) they are linearly independent, (ii) they span V.
0
e =
1 ,
F
0
i F
.
..
F
0F
U = {u1 , u2 , . . . , uk , . . .} V
0 v + 1 u1 + + n un = 0.
v = 1
0 (1 u1 + + n un ),
! From Theorem 508 it follows that to shew that a vector space has a basis it is
enough to shew that it has a maximal linearly independent set of vectors. Such a
proof requires something called Zrns Lemma, and it is beyond our scope. We dodge
the whole business by taking as an axiom that every vector space possesses a basis.
w1 = 1 u1 + 2 u2 + + n un
u1 = 1
1 (w1 (2 u2 + + n un )),
Bases 155
Assume now that the theorem is true for any set of fewer than k inde-
pendent vectors. We may thus assume that that {u1 , . . .} has more than
k 1 vectors and that
Since wk U we have
uk = 1
k (wk (1 w1 + 2 w2 + + k1 wk1 + k+1 uk+1 + m um )).
{x1 , x2 , . . . , xk }
Proof Take any basis {u1 , . . . , uk , uk+1 , . . . , } and use Steinitz Replacement
Theorem 509.
514 Example Find a basis and the dimension of the space generated
by the set of symmetric matrices in Mn (R).
Solution: Let Eij Mn (R) be the n n matrix with a 1 on the ij-th position
and 0s everywhere else. For 1 i < j n, consider the n 2 = 2
"#
n(n 1)
matrices Aij = Eij + Eji . The Aij have a 1 on the ij-th and ji-th position
and 0s everywhere else. They, together with the n matrices Eii , 1 i n
constitute a basis for the space of symmetric matrices. The dimension of
this space is thus
n(n 1) n(n + 1)
+n= .
2 2
x 1
x
that the us are linearly independent. But if X = . , then
..
2
xn
x1 u1 + + xn un = AX.
AX = 0n = P1 Dn,n,r Q1 X = 0n = Dn,n,r Q1 X = 0n .
y
1
y
X = . . Then
..
2
Put Q1
yn
Dn,n,r Q1 X = 0n = y1 e1 + + yr er = 0n ,
where ej is the n-dimensional column vector with a 1 on the j-th slot and
0s everywhere else. If r < n then yr+1 , . . . , yn can be taken arbitrarily
and so there would not be a unique solution, a contradiction. Hence
r = n and A is invertible.
X= 5b : a, b R
a + 2b
a
{v1 + v2 , v2 + v3 , v3 + v4 , v4 + v5 , v5 + v1 }
518 Problem Find a basis for the solution space of the system of n + 1
linear equations of 2n unknowns
x1 + x2 + + xn = 0,
x2 + x3 + + xn+1 = 0,
......
...
xn+1 + xn+2 + + x2n = 0.
X = {(a, b, c, d)|b + 2c = 0} R4
520 Problem Prove that the dimension of the vector subspace of lower
n(n + 1)
triangular n n matrices is and find a basis for this space.
2
+
1 0 1
+
1 0 2
Coordinates 159
523 Problem Find a basis and the dimension of
1 0,
X = span
v = v2
1
=
0
, v3
0 1, v = 1
=
1
.
0
1 4
0 1 2 0 2 0 0
ax = b,
for x.
5.6 Coordinates
525 Theorem Let {v1 , v2 , . . . , vn } be a basis for a vector space V. Then
any v V has a unique representation
v = a1 v1 + a2 v2 + + an vn .
Proof Let
v = b1 v1 + b2 v2 + + bn vn
be another representation of v. Then
a1 b1 = a2 b2 = = an bn = 0F ,
that is
a1 = b1 ; a2 = b2 ; ; an = bn ,
proving uniqueness.
v = a1 v1 + a2 v2 + + an vn .
527 Example The standard ordered basis for R3 is S = {i, j, k}. The vector
1
2 R
for example, has coordinates (1, 2, 3)S . If the order of the basis
3
3
1
were changed to the ordered basis S1 = {i, k, j}, then 2 R
3
would
3
have coordinates (1, 3, 2)S1 .
1
528 Example Consider the vector 2 R 3
(given in standard represen-
3
tation). Since
1 1 1 1
2 = 1 0 1 1 + 3 1 ,
3 0 0 1
1 1 1
1
2 has coordinates
under the ordered basis B1 =
0
0
, 1 , 1
,
0 1 3
(1, 1, 3)B1 . We write
1 1
2 = 1
.
3 3
B1
Coordinates 161
B1
1
= ,
1
1 2
Thus
x 2 1 1 1 3
1
=
y 1 1 1 2 4
B2
=
1 1 3
1
3
1
3
1 2
3 31 2 4
=
1 3
2
3
1
3 1 4
=
6 .
5
B2
6 2 1 7
= 6 5 = .
5 1 1 11
B2
Also,
XB1 = A1 BYB2 .
1 1 2
Find the transition matrix from B1 to B2 and also the transition matrix
1
from B2 to B . Also find the coordinates of
1 2 in terms of B2 .
3
B1
Coordinates 163
Solution: Let
1 1 1 1 1 2
A= 1 0 B= 0
1
, 1 1
.
1 0 0 1 0 0
P = B1 A
1
1 1 2 1 1 1
1 1 0 1 1 0
=
1 0 0 1 0 0
0 0 1 1 1 1
0 1 1 1 1 0
=
1
1
2 2 1 1 0 0
1 0 0
2 1 0 .
=
1
2 1 2
1
1 0 0 1 0 0
= 2 0 = 0
P1 1
1
2 1
.
2 1 2 0 2 2
Now,
1 0 0 1
2 1
= 2 0 = 4
YB2 1
1
11
.
2 1 2 3 2
B1 B2
164 Chapter 5
B1 = , , ,
1 1 1 0
0
1 1 1
and
0 2 1
2
1 1 3
= , , ,
1
B2
0 2 1 1
1 2 2 2
Solution: Let
1 1 1 1
2 0 2 1
2 1 , B = 1 3
A=
1 2 1 1
1 0
1 1
0 2 1 1
0 1 1 1 1 2 2 2
AP = B,
Coordinates 165
which gives
P = A1 B.
Using elementary row operations we find
3/13 2/13 6/13 5/13
5/13 4/13
=
1/13 3/13
A1
2/13 1/13
3/13 4/13
3/13 2/13 7/13 8/13
and so
3/13 2/13 6/13 5/13
2 0 2 1
1 0 0 1
5/13 4/13 1 3 = 1 1
P=
1/13 3/13 1 1 1 0
2/13
1/13 0
1 0 1
3/13 4/13
2 1
1 1
3/13 2/13 7/13 8/13 1 2 2 2 0 0 1 0
533 Problem 1. Prove that the following vectors are linearly indepen-
dent in R4
1 1 1 1
1 1 1 1
= , = , = , = .
a1
1 a2
1 a3
1 a4
1
1 1 1 1
1
2
2. Find the coordinates of under the ordered basis (a , a , a , a ).
1 1 2 3 4
1
1
2
3. Find the coordinates of under the ordered basis (a , a , a , a ).
1 1 3 2 4
1
166 Chapter 5
1 1
and
0
1 0
B2 =
1
1
,
1 , 0
1 1
are bases for R3 . Find the transition matrix from B1 to B2 and then find
1
the coordinates of 2 under B2 .
0
B1
Chapter 6
Linear Transformations
V W
L: ,
a 7 L(a)
is a function which is
! It is clear that the above two conditions can be summarised conveniently into
167
168 Chapter 6
538 Example For a point (x, y) R2 , its reflexion about the y-axis is (x, y).
Prove that
R2 R2
R:
(x, y) 7 (x, y)
is linear.
= ((x1 + x2 ), y1 + y2 )
= (x1 , y1 ) + (x2 , y2 )
whence R is linear.
Solution: Since
5= 4 1 1,
3 1 1
Linear Transformations 169
we have
1 2 6
5 1 1 1 0 4
L = 4L L = 4 = .
2 2 6
3 1 1
3 3 9
540 Theorem Let hV, +, , Fi and hW, +, , Fi be vector spaces over the
same field F, and let L : V W be a linear transformation. Then
L(0V ) = 0W .
x V, L(x) = L(x).
Proof We have
L(0V ) = L(0V + 0V ) = L(0V ) + L(0V ),
hence
L(0V ) L(0V ) = L(0V ).
Since
L(0V ) L(0V ) = 0W ,
we obtain the first result.
Now
0W = L(0V ) = L(x + (x)) = L(x) + L(x),
from where the second result follows.
R3 R3 R3
L:
(x, y) 7 xk + hy
is a linear transformation.
170 Chapter 6
is a linear transformation.
544 Problem Let V be a vector space and let S V. The set S is said to
be convex if [0; 1], x, y S, (1 )x + y S, that is, for any two
points in S, the straight line joining them also belongs to S. Let T : V W
be a linear transformation from the vector space V to the vector space
W. Prove that T maps convex sets into convex sets.
ker (T ) = {v V : T (v) = 0W }.
546 Theorem Let hV, +, , Fi and hW, +, , Fi be vector spaces over the
same field F, and
V W
T:
v 7 T (v)
be a linear transformation. Then ker (T ) is a vector subspace of V and
Im (T ) is a vector subspace of W.
Now, let (w1 , w2 ) (Im (T ))2 and F. Then (v1 , v2 ) V 2 such that
T (v1 ) = w1 and T (v2 ) = w2 . We must prove that w1 + w2 Im (T ), that
is, that v such that T (v) = w1 + w2 . But
w1 + w2 = T (v1 ) + T (v2 ) = T (v1 + v2 ),
and so we may take v = v1 + v2 . This proves that Im (T ) is a subspace
of W.
547 Theorem Let hV, +, , Fi and hW, +, , Fi be vector spaces over the
same field F, and
V W
T:
v 7 T (v)
be a linear transformation. Then T is injective if and only if ker (T ) = 0V .
Conversely, assume that ker (T ) = {0V }, and that T (x) = T (y). We must
prove that x = y. But
= T (x y) = 0W
= (x y) ker (T )
= x y = 0V
= x = y,
as we wanted to shew.
V W
T:
v 7 T (v)
172 Chapter 6
Hence
n
X n
X
w = T (v) = i T (vi ) = i T (vi ),
i=1 i=k+1
X
n
X n
0W =
i=k+1
i T (vi ) = T
i=k+1
i vi
.
Pn
This means that i=k+1 i vi ker (T ), which is impossible unless k+1 =
k+2 = = n = 0F .
Solution: Put A =
a band assume L(A) = 0 . Then 2
c d
0 0= L(A) = a c a b= (c b) 0 1.
0 0 b d c d 1 0
ker (L) =
: (a, b, d) R .
b
a
3
b d
551 Problem In problem 541 we saw that L : R3 R3 ,
x x y z
L y = x + y + z
z z
1 2 1
satisfy
x + 2y
553 Problem It is easy to see that L : R2 R3 ,
x x + 2y
L =
y
0
174 Chapter 6
x y
x x + y
L =
y
0
M2 (R) M2 (R)
L:
A 7 AT + A
is a linear transformation.
2. Find a basis for ker (L) and find dim ker (L)
a n1
A
a 12
a
a12 w1 + a22 w2 + + an2 wn ..
22
.
L(v2 ) = =
.
an2
A
.. .. .. .. ..
. . . .
.
a 1n
a
L(vm ) = a1m w1 + a2m w2 + + anm wn .. 2n
.
=
anm
A
ML .. .. ..
. . .
Solution:
1 1
1. Solution: The matrix will be a 3 3 matrix. We have L 0 = 1,
0 0
1 1 1
1 1 1 .
0 0 1
0 0 0 0 1 0
A
1 0 1 1 1 2
L 1= 2= 2 0+ 2 1+ 0 0= 2 ,
0 0 0 0 1 0
A
Matrix Representation 177
and
1 0 1 1 1 3
L 0= 2= 3 0+ 2 1+ 1 0= 2 ,
1 1 0 0 1 1
A
560 Example Let Rn [x] denote the set of polynomials with real coeffi-
cients with degree at most n.
1. Prove that
R3 [x] R1 [x]
L:
p(x) 7 p 00 (x)
2. Find the matrix of L using the ordered bases {1, x, x2 , x3 } for R3 [x]
and {1, x} for R2 [x].
3. Find the matrix of L using the ordered bases {1, x, x2 , x3 } for R3 [x]
and {1, x + 2} for R1 [x].
4. Find a basis for ker (L) and find dim ker (L).
Solution:
whence L is linear.
178 Chapter 6
2. We have
L(1) =
d 2
1 = 0 = 0(1) + 0(x) =
0,
dx2
0
L(x) =
d2
x = 0 = 0(1) + 0(x) =
0,
dx2
0
L(x2 ) =
d2 2
x = 2 = 2(1) + 0(x) =
2,
dx2
0
L(x3 ) =
d2 3
x = 6x = 0(1) + 6(x) =
0,
dx2 6
whence the matrix representation of L under the standard basis is
0 0 2 0
.
0 0 0 6
3. We have
L(1) =
d2
1 = 0 = 0(1) + 0(x + 2) =
0,
dx2
0
L(x) =
d2
x = 0 = 0(1) + 0(x + 2) =
0,
dx2
0
L(x2 ) =
d2 2
x = 2 = 2(1) + 0(x + 2) =
2,
dx2
0
L(x3 ) =
d2 3
x = 6x = 12(1) + 6(x + 2) =
12,
dx2 6
whence the matrix representation of L under the standard basis is
0 0 2 12
.
0 0 0 6
Matrix Representation 179
561 Example
1. A linear transformation T : R3 R3 is such that
2 3
T (i) = 1; T (j) = 0 .
1 1
It is known that
Im (T ) = span (T (i), T (j))
and that
1
2
ker (T ) = span
.
1
Argue that there must be and such that
T (k) = T (i) + T (j).
2. Find and , and hence, the matrix representing T under the stan-
dard ordered basis.
Solution:
1. Since T (k) Im (T ) and Im (T ) is generated by T (i) and T (k) there
must be (, ) R2 with
2 3 2 + 3
T (k) = T (i) + T (j) = 1+ 0 = .
1 1
180 Chapter 6
2. The matrix of T is
2 3 2 + 3
T (i) T (k) = 1
T (j) 0
.
1 1
1
Since 2 ker (T ) we must have
1
2 3 2 + 3 1 0
1 0 2 = 0 .
1 1 1 0
Find a, b, c.
1. Prove that T : R2 R3
x + y
563 Problem
x x y
T =
y
2x + 3y
is a linear transformation.
2
3
1 1 0
R and B =
of R .
2 3
1 , 0 , 1
1 1
0
where
x x + 2y
L y= 3x z .
z
1. The bases for both R3 and R2 are both the standard ordered bases.
1 1 1
2. The ordered basis for R3 is 0, 1, 1and R
2
has the standard
0 0 1
ordered basis .
182 Chapter 6
1 1 1
3. The ordered basis for R3 is 0, 1, 1and the ordered basis for
0 0 1
R2
1
is A = , .
1
0 1
565 Problem A linear transformation T : R2 R2 satisfies ker (T ) = Im (T ),
1 2
and T = . Find the matrix representing T under the standard
1 3
ordered basis.
566 Problem Find the matrix representation for the linear map
M2 (R) R
L: ,
A 7 tr (A)
under the standard basis
A =
1 0
,
0 1
,
0 0
,
0 0
0 0
0 0 1 0 0 1
for M2 (R).
567 Problem Let A Mnp (R), B Mpq (R), and C Mqr (R), be such
that rank (B) = rank (AB). Shew that
7.1 Permutations
568 Definition Let S be a finite set with n 1 elements. A permutation is
a bijective function : S S. It is easy to see that there are n! permuta-
tions from S onto itself.
Since we are mostly concerned with the action that exerts on S rather
than with the particular names of the elements of S, we will take S to be
the set S = {1, 2, 3, . . . , n}. We indicate a permutation by means of the
following convenient diagram
1
=
2 n
.
(1) (2) (n)
569 Definition The notation Sn will denote the set of all permutations on
{1, 2, 3, . . . , n}. Under this notation, the composition of two permutations
(, ) S2n is
=
1 2 n
1 2 n
(1) (2) (n) (1) (2)
(n)
.
=
1 2 n
( )(1) ( )(2) ( )(n)
183
184 Chapter 7
} = k .
| {z
k compositions
1 : S S
then
1
(1)
=
(2) (n)
.
1 2 n
2 3
570 Example The set S3 has 3! = 6 elements, which are given below.
1. Id : {1, 2, 3} {1, 2, 3} with
1
Id =
2 3
.
1 2 3
Permutations 185
Id 1 2 3 1 2
Id Id 1 2 3 1 2
1 1 Id 1 2 2 3
2 2 2 Id 1 3 1
3 3 1 2 Id 1 2
2 2 2 3 1 Id 1
1 1 3 1 2 2 Id
Observe that 1
1 = 1 .
Observe that 1
1 = 2 .
the idea of a cycle. Consider in S9 the permutation
1
=
2 3 4 5 6 7 8 9
.
2 1 3 6 9 7 8 4 5
! Observe that (i2 . . . il i1 ) = (i1 . . . il ) etc., and that (1) = (2) = = (n) = Id .
In fact, we have
(i1 . . . il ) = (j1 . . . jm )
if and only if (1) l = m and if (2) l > 1: a such that k: ik = jk+a mod l . Two cycles
(i1 , . . . , il ) and (j1 , . . . , jm ) are disjoint if {i1 , . . . , il } {j1 , . . . , jm } = . Disjoint cycles
commute and if = 1 2 t is the product of disjoint cycles of length l1 , l2 , . . . , lt
respectively, then has order
lcm (l1 , l2 , . . . , lt ) .
575 Example A cycle decomposition for S9 ,
1
=
2 3 4 5 6 7 8 9
1 8 7 6 2 3 4 5 9
188 Chapter 7
is
(285)(3746).
The order of is lcm (3, 4) = 12.
Solution: Putting the top card at the bottom corresponds to
1 2 3 4 5 6 7 8 9 10
.
2 3 4 5 6 7 8 9 10 1
Cutting this new arrangement in half and putting the lower half on top
corresponds to
1 2 3 4 5 6 7 8 9 10
.
7 8 9 10 1 2 3 4 5 6
Cycle Notation 189
(23468) = (23)(34)(46)(68).
sgn() = sgn()sgn().
Proof We have
Q ()(i)()(j)
sgn() =
Q .
1i<jn ij
= Q 1i<jn
((i))((j))
(i)(j) 1i<jn
(i)(j)
ij
The second factor on this last equality is clearly sgn(), we must shew
that the first factor is sgn(). Observe now that for 1 a < b n we
have
(a) (b) (b) (a)
= .
ab ba
Since and are permutations, b 6= a, (i) = a, (j) = b and so (i) =
(a), (j) = b. Thus
((i)) ((j)) (a) (b)
=
(i) (j) ab
and so
Y ((i)) ((j)) Y (a) (b)
= = sgn().
(i) (j) ab
1i<jn 1a<bn
sgn() = (1)l1
192 Chapter 7
Proof This follows at once from Theorem 586 and Lemma 589.
591 Example The cycle (4678) is an odd cycle; the cycle (1) is an even
cycle; the cycle (12345) is an even cycle.
Proof This follows from Theorem 579, Lemma 582, and Corollary 590.
5 6 7 8
(7.1)
9 10 11 12
13 14 15
Cycle Notation 193
In this grid we may successively exchange the empty slot with any of its
neighbours, as for example
1 2 3 4
5 6 7 8
. (7.2)
9 10 11 12
13 14 15
5 6 7 8
(7.3)
9 10 11 12
13 15 14
Solution: Let us shew that this is impossible. Each time we move a square
to the empty position, we make transpositions on the set {1, 2, . . . , 16}.
Thus at each move, the permutation is multiplied by a transposition and
hence it changes sign. Observe that the permutation corresponding to
the square in (7.3) is (14 15) (the positions 14th and 15th are transposed)
and hence it is an odd permutation. But we claim that the empty slot
can only return to its original position after an even permutation. To see
this paint the grid as a checkerboard:
B R B R
R B R B
(7.4)
B R B R
R B R B
We see that after each move, the empty square changes from black to
red, and thus after an odd number of moves the empty slot is on a red
square. Thus the empty slot cannot return to its original position in an
odd number of moves. This completes the proof.
594 Problem Decompose the permutation
1 2 3 4 5 6 7 8 9
2 3 4 1 5 8 6 7 9
194 Chapter 7
7.3 Determinants
There are many ways of developing the theory of determinants. We
will choose a way that will allow us to deduce the properties of de-
terminants with ease, but has the drawback of being computationally
cumbersome. In the next section we will shew that our way of defining
determinants is equivalent to a more computationally friendly one.
then
det A = sgn(Id )a1Id (1) a2Id (2) + sgn()a1(1) a2(2) = a11 a22 a12 a21 .
598 Example From the above formula for 2 2 matrices it follows that
det A =
1 2
det
3 4
= (1)(4) (3)(2) = 2,
det B =
1 2(1)(4) (3)(2)
det
3 4
= 10,
and
0 4= (0)(8) (6)(4) = 24.
det(A + B) = det
6 8
then
det A = sgn(Id )a1Id (1) a2Id (2) a3Id (3) + sgn(1 )a11 (1) a21 (2) a31 (3)
+sgn(2 )a12 (1) a22 (2) a32 (3) + sgn(3 )a13 (1) a23 (2) a33 (3)
+sgn(1 )a11 (1) a21 (2) a31 (3) + sgn(2 )a12 (1) a22 (2) a32 (3)
Then (a) = (a) for a {1, 2, . . . , n}\{s, t}. Also, sgn() = sgn()sgn() =
sgn(). As ranges through all permutations of Sn , so does , hence
P
det B = Sn sgn()b1(1) b2(2) bs(s) bt(t) bn(n)
P
= Sn sgn()a1(1) a2(2) ast ats an(n)
P
= Sn sgn()a1(1) a2(2) as(s) at(t) an(n)
P
= Sn sgn()a1(1) a2(2) an(n)
= det A.
..
.
A(r:(n))
det AT = det A.
det AT = det C
P
= Sn sgn()c1(1) c2(2) cn(n)
P
= Sn sgn()a(1)1 a(2)2 a(n)n .
But the product a(1)1 a(2)2 a(n)n also appears in det A with the same
signum sgn(), since the permutation
(1) (2) (n)
1 2 n
Proof This follows upon combining Theorem 600 and Theorem 602.
det C = det A.
Proof This follows upon using Theorem 602 and Theorem 604.
! It follows from Theorem 604 and Corollary 605 that if a row (or column) of a matrix
consists of 0F s only, then the determinant of this matrix is 0F .
606 Example
x 1 a 1 1 a
det x b b
= x det x .
2
1 1
x3 1 c x2 1 c
607 Corollary
det(A) = n det A.
Proof Since there are n columns, we are able to pull out one factor of
from each one.
Solution: We have
a11 a12 a1n
a21 a22 a2n
. . . .
. . . .
det
. . . .
a(s+1)1 a(s+1)2 a(s+1)n
. . . .
. . . .
. . . .
a21 a22 a2n
. . . .
. . . .
= det
. . . .
a(s+1)1 a(s+1)2 a(s+1)n
. . . .
. . . .
. . . .
a21 a22 a2n
.
. . . .
. . . .
+ det
. . . .
a(s+1)1 a(s+1)2 a(s+1)n
. . . .
. . . .
. . . .
Proof Put
a11 a12 a1n
a21 a22 a2n
,
. . . .
. . . .
S=
. . . .
a(s+1)1 a(s+1)2 a(s+1)n
. . . .
. . . .
. . . .
a11 a12 a1n
a21 a22 a2n
. . . .
. . . .
T=
. . . .
a(s+1)1 a(s+1)2 a(s+1)n
. . . .
. . . .
. . . .
and
a11 a12 a1n
a21 a22 a2n
.
. . . .
. . . .
U=
. . . .
a(s+1)1 a(s+1)2 a(s+1)n
. . . .
. . . .
. . . .
Then
P
det S = S n sgn()a1(1) a2(2) a(s1)(s1) (bs(s)
= det T + det U.
610 Lemma If two rows or two columns of A Mn (F), A = [aij ] are iden-
tical, then det A = 0F .
Proof Suppose asj = atj for s 6= t and for all j [1; n]. In particular, this
means that for any Sn we have as(t) = at(t) and at(s) = as(s) .
Let be the transposition
s
=
t
.
(t) (s)
Then (a) = (a) for a {1, 2, . . . , n}\{s, t}. Also, sgn() = sgn()sgn() =
sgn(). As runs through all even permutations, runs through all odd
Determinants 201
=
P
Sn sgn()a 1(1) a2(2) as(s) at(t) an(n)
sgn()=1
=
P
Sn
sgn() a1(1) a2(2) as(s) at(t) an(n)
sgn()=1
=
P
Sn
sgn() a1(1) a2(2) as(s) at(t) an(n)
sgn()=1
= 0F .
Proof Suppose asj = atj for s 6= t and for all j [1; n]. If B is the matrix ob-
tained by pulling out the factor from the s-th row then det A = det B.
But now the s-th and the t-th rows in B are identical, and so det B = 0F .
Arguing on AT will yield the analogous result for the columns.
612 Example
1 a b 1 1 b
det 1 c c
a
= a det 1 1
= 0,
1 a d 1 1 d
since on the last determinant the first two columns are identical.
Proof For the row transvection it suffices to take bsj = asj , csj = atj for
j [1; n] in Lemma 609. With the same notation as in the lemma, T = A,
and so,
det X = det T + det U = det A + det U.
But U has its s-th and t-th rows proportional (s 6= t), and so by Corollary
611 det U = 0F . Hence det X = det A. To obtain the result for column
transvections it suffices now to also apply Theorem 602.
Solution: Observe that 299, 468 and 741 are all divisible by 13. Thus
7 4 1 7 4 741 7 4 57
= sgn(Id )a1Id (1) a2Id (2) anId (n) = a11 a22 ann .
Solution: We have
1 2 3 1 0 0
det 6 det 3 6
C2 2C1 C2
4 5
C3 3C1 C3
4
7 8 9 7 6 12
1 0 0
(3)(6) det 4 1 1
=
7 2 2
= 0,
since in this last matrix the second and third columns are identical and
so Lemma 610 applies.
Pn
Proof Put D = AB, D = (dij ), dij = k=1 aik bkj . If A(c:k) , D(c:k) , 1 k n
denote the columns of A and D, respectively, observe that
n
X
D(c:k) = blk A(c:l) , 1 k n.
l=1
By Lemma 610, if any two of the A(c:jl ) are identical, the determinant on
the right vanishes. So each one of the jl is different in the non-vanishing
204 Chapter 7
{1, 2, . . . , n} {1, 2, . . . , n}
:
l 7 jl
= (sgn()) det A.
We deduce that
det(AB) = det D
Pn
= jn =1 b1j1 b2j2 bnjn det(A(c:j1 ) , A(c:j2 ) , . . . , A(c:jn ) )
P
= (det A) Sn (sgn())b1(1) b2(2) bn(n)
as we wanted to shew.
a3 b3 c3
2c 2c cab
Determine the numerical values of det A1 , det A2 , det A3 , det A4 and det A5 .
such that
A = X + Y.
That is, any square matrix over R can be written as a sum of two matrices
whose determinant is not zero.
Put X
Cij = (sgn())a1(1) a2(2) an(n) .
Sn
(i)=j
Then
P
det A = Sn (sgn())a1(1) a2(2) an(n)
Pn P
= i=1 aij Sn (sgn())a1(1) a2(2)
(i)=j (7.5)
a(i1)(i1) a(i+1)(i+1) an(n)
Pn
= i=1 aij Cij ,
is the expansion of det A along the j-th column. Similarly,
P
det A = Sn (sgn())a1(1) a2(2) an(n)
Pn P
= j=1 aij Sn (sgn())a1(1) a2(2)
(i)=j
627 Definition Let A Mn (F), A = [aij ]. The ij-th minor Aij Mn1 (R) is
the (n 1) (n 1) matrix obtained by deleting the i-th row and the j-th
column from A.
628 Example If
1 2 3
A= 4 6
5
7 8 9
then, for example,
A11
5 6,
= A12
4 6,
= A21
2 3,
= A22
1 3,
= A33
1 2.
=
8 9 7 9 8 9 7 9 4 5
= det Ann ,
since the second sum shewn is the determinant of the submatrix ob-
tained by deleting the last row and last column from A.
To find Cij for general ij we perform some row and column inter-
changes to A in order to bring aij to the nn-th position. We thus bring
the i-th row to the n-th row by a series of transpositions, first swapping
the i-th and the (i + 1)-th row, then swapping the new (i + 1)-th row and
the (i + 2)-th row, and so forth until the original i-th row makes it to the
n-th row. We have made thereby n i interchanges. To this new matrix
we perform analogous interchanges to the j-th column, thereby making
n j interchanges. We have made a total of 2n i j interchanges.
Observe that (1)2nij = (1)i+j . Call the analogous quantities in the
resulting matrix A 0 , Cnn
0 0
, Ann . Then
0
Cij = Cnn = det Ann
0
= (1)i+j det Aij ,
by virtue of Corollary 601.
square matrix. We always obtain the same result. The sign pattern is given by
+ +
.
.
+ + .
.
.
+ + .
.. .. .. .. ..
. . . . .
Solution: We have
det A = 1(1)1+1
5 6+ 2(1)
det 1+2 4 6+ 3(1)
det 1+3 4 5
det
8 9 7 9 7 8
= 1(45 48) 2(36 42) + 3(32 35) = 0.
Solution:
1 1 1 1 0 0
det a c
b
= det a
b a c a
a2 b2 c2
a b a c a
2 2 2 2 2
= det
b a c a
2
b c c a 2
2
2
=
(b a)(c a) det
1 1
b+a c+a
3 2 1 1997
2000 1999 1998 1997 1
Laplace Expansion 209
1
.
1 1 1 1
det 1 1
1 1 1 1
1 1 1 1 1 1
2000 1999 1998 1997 2 1
1
.
0 0 2 2
det 0 1
0 0 0 2
0 0 0 0 0 1
2001 2000 1999 1998 3 1
1
1998
).
0 0 2
0 0 0 0 1
Proof We have
Pn
[A(adj (A))]ij = k=1 aik [adj (A)]kj
Pn
= k=1 aik (1)i+k det Ajk .
638 Problem If
1 1 1 1
x 0= 0,
det
a 0
x 0
0 b
x 0 0 c
1 1 1 1
= + + .
x a b c
0 d
0 c 0 d
0
n+1
.
. 0 1 0
..
.. .. ..
. . .
0 0 0 1 0
212 Chapter 7
. n n 4 n
..
.. .. ..
. . .
n n n n n n
that is, A Mn (R), A = [aij ] is a matrix such that akk = k and aij = n
when i 6= j. Find det A.
,n ,n 1 ,n 1
k- k 1- k -
= + .
Prove that
# # #
1 " " " 1
1 1 " # " # " #
n n n
1 2 n1
#
n n n
" " n n
n n n
n1 1 1 n3 n2
# # #
n
1 " " " 1 1
n
2
n
3
644 Problem Let A GLn (F), n > 1. Prove that det(adj (A)) = (det A)n1 .
adj SAS1 = S(adj (A))S1 .
646 Problem If A GL2 (F), , and let k be a positive integer. Prove that
det(adj adj(A)) = det A.
| {z }
k
Determinants and Linear Systems 213
A is invertible.
B = PAP1 .
215
216 Chapter 8
Suppose that
1 0 0 0
0 0 .
A = . ..
2 0
.. .. ..
.
. .
0 0 0 n
Then if K is a positive integer
K
0 0 0
0 0
1
= .
K 0
..
.
..
2
AK
.
.. ..
. .
0 0 0 K
n
= P .
K 0
..
P
..
2
BK = (PAP1 )(PAP1 ) (PAP1 ) = PAK P1 1
.
.. .. ,
| {z }
K factors
. .
0 0 0 K
n
from where it follows that APk = k Pk . This motivates the following defini-
tion.
det(In A) = 0F
1 1 0 0
1 0. Find
656 Example Let A =
1 0
0 1
0 1
0 0 1 1
Solution: We have
218 Chapter 8
1 1 0 0
1 0
det
1 0
det(I4 A) =
0 1
0 1
0
0 1 1
1 0 0
1 0 0
( 1) det 0 1 1
= 1
+ det 0 1
0 1 1 0 1 1
= ( 2)()(( 1)2 1)
= ( 2)2 ()2
If = 0, then
1 1 0 0
1 0 .
1 0
0 1
0I4 A =
0 1
0 0 1 1
and if
Eigenvalues and Eigenvectors 219
1 1 0 0
a 0
0 1 b = 0 ,
0 1
0 0 0 0 c 0
0 0 0 0 d 0
then c = d and a = b
Thus the general solution of the system (0I4 A)X = 0n1 is
a 1 0
b 1 0
= a + c .
c 0 1
d 0 1
If = 2, then
1 1 0
0
1 0 .
1 0
0 1
2I4 A =
0 1
0 0 1 1
and if
1 1 0 0
a 0
0 1 b = 0 ,
0 1
0 0 0 0 c 0
0 0 0 0 d 0
220 Chapter 8
then c = d and a = b
Proof Put p() = det(In A). Then p(0F ) = det(A) = (1)n det A is the
constant term of the characteristic polynomial. If = 0F is an eigenvalue
then
p(0F ) = 0F = det A = 0F ,
Proof We have
1
659 Problem Find the eigenvalues and eigenvectors of A =
1
1 1
0 2 1
660 Problem Let A = 2 2
3
. Find
1 2 0
The eigenvalues of A.
8.3 Diagonalisability
In this section we find conditions for diagonalisability.
. .
0 0 0 n
Diagonalisability 223
where the Pk are the columns of P. Since P is invertible, the Pk are linearly
independent by virtue of Theorems 474 and 647.
A = PDP1 .
We may take
2 0 0 1 1 4
D= 0 0 P=
2
, 1 0 1
.
0 0 3 1 1 1
We also find
1
1 1
= 1
5 5
P1 0 1
.
1 1
5 0 5
664 Problem Let A be a 2 2 matrix with eigenvalues 1 and 2 and cor-
1 1
responding eigenvectors and , respectively. Determine A 10
.
0 1
( + 1)2 ( 3).
1 1
One of the eigenvalues has two eigenvectors 0 and 1. The other
0 0
1
eigenvalue has corresponding eigenvector 1. Determine A.
1
Diagonalisability 225
1. Find det A.
2. Find A1 .
3. Find rank (A I4 ).
4. Find det(A I4 ).
8. Find A10 .
667 Problem Let U Mn (R) be a square matrix all whose entries are
equal to 1.
2. Find det A.
5. Shew that
n 0 0
0 0 P
U = P . ..
0
.. .. ..
.
1
,
. .
0 0 0
226 Chapter 8
where
1 1 0 0 0
1 0
0 1 0
..
P=
1 ..
.
..
. .
.
... ..
0 0
.
.. .. .. ..
. . . .
1 1
0 0 0
1 1 1 1 1
Appendix A
Some Answers and Hints
17 Solution
x X \ (X \ A) x X x 6 (X \ A)
xXxA
x X A.
18 Solution
X \ (A B) x X (x 6 (A B))
x X (x 6 A x 6 B)
(x X x 6 A) (x X x 6 B)
x (X \ A) x (X \ B)
x (X \ A) (X \ B).
A B C = A (B \ A) (C \ (A B)).
23 Solution We have
|a| = |a b + b| |a b| + |b|,
giving
|a| |b| |a b|.
Similarly,
|b| = |b a + a| |b a| + |a| = |a b| + |a|,
227
228 Appendix A
gives
|b| |a| |a b|.
The stated inequality follows from this.
a a b
= = mn Z,
c b c
and so a c.
2
39 Solution Here is one possible example: put a b a b+a Z. Then
2
clearly if a Z \ {0} we have a a since a a+a = a + 1 Z. On the other
2
hand, the relation is not symmetric, since 5 2 as 5 2+5 = 15 Z but 2 6 5,
2 2
as 2 5+2 = 65 6 Z. It is not transitive either, since 5 3+5 Z = 5 3 and
32 +3 52 +5
12 Z = 3 12 but 12 6 Z and so 5 12.
1
41 Answer [B] [x] = x + Z. [C] No.
3
55 Solution Let = 12 + i 2 .
3
Then 2 + + 1 = 0 and 3 = 1. Then
(a + b + c)(u + v + w) = au + av + aw + bu + bv + bw + cu + cv + cw,
(a + b + 2 c)(u + v + 2 w) = au + bw + cv
+(av + bu + cw)
+2 (aw + bv + cu),
and
(a + 2 b + c)(u + 2 v + w) = au + bw + cv
+(aw + bv + cu)
+2 (av + bu + cw).
Some Answers and Hints 229
56 Solution We have
x>(y>z) = (x>y)>z,
associativity is established.
a+b
1 < <1 1 ab < a + b < 1 + ab
1 + ab
1 ab a b < 0 < 1 + ab a b
a+b b+a
Since a b = = = b a, commutativity follows trivially.
1 + ab 1 + ba
Now
a (b c) = a
b+c
1 + bc
!
a+
b+c
1 + bc
!
!
=
b+c
1+a
1 + bc
a(1 + bc) + b + c a + b + c + abc
= = .
1 + bc + a(b + c) 1 + ab + bc + ca
(a b) c =
a+b
1 + ab
! c
a+b
1 + ab
!+c
!
=
a+b
1+ c
1 + ab
(a + b) + c(1 + ab)
=
1 + ab + (a + b)c
a + b + c + abc
= ,
1 + ab + bc + ca
whence is associative.
a+e
If ae = a then = a, which gives a+e = a+ea2 or e(a2 1) = 0.
1 + ae
Since a 6= 1, we must have e = 0.
a+b
If a b = 0, then = 0, which means that b = a.
1 + ab
a (b c) = a (b + c bc)
= a + b + c bc a(b + c bc)
= a + b + c ab bc ca + abc.
Some Answers and Hints 231
(a b) c = (a + b ab) c
= a + b ab + c (a + b ab)c
= a + b + c ab bc ca + abc,
whence is associative.
59 Solution We have
x y = (x y) (x y)
= [y (x y)] x
= [(x y) x] y
= [(y x) x] y
= [(x x) y] y
= (y y) (x x)
= y x,
proving commutativity.
x2 = 5.
2 2 2 2 2
Now, the squares modulo 11 are 0 = 0, 1 = 1, 2 = 4, 3 = 9, 4 = 5,
2 2
5 = 3. Also, (11 4)2 = 7 = 5. Hence the solutions are x = 4 or x = 7.
232 Appendix A
+ 0 1 2 3 4 5 6 7 8 9 10
0 0 1 2 3 4 5 6 7 8 9 10
1 1 2 3 4 5 6 7 8 9 10 0
2 2 3 4 5 6 7 8 9 10 0 1
3 3 4 5 6 7 8 9 10 0 1 2
4 4 5 6 7 8 9 10 0 1 2 3
5 5 6 7 8 9 10 0 1 2 3 4
6 6 7 8 9 10 0 1 2 3 4 5
7 7 0 9 10 0 1 2 3 4 5 6
8 8 9 10 0 1 2 3 4 5 6 7
9 9 10 0 1 2 3 4 5 6 7 8
10 10 0 1 2 3 4 5 6 7 8 9
0 1 2 3 4 5 6 7 8 9 10
0 0 0 0 0 0 0 0 0 0 0 0
1 0 1 2 3 4 5 6 7 8 9 10
2 0 2 4 6 8 10 1 3 5 7 9
3 0 3 6 9 1 4 7 10 2 5 8
4 0 4 8 1 5 9 2 6 10 3 7
5 0 5 10 4 9 3 8 2 7 1 6
6 0 6 1 7 2 8 3 9 4 10 5
7 0 7 3 10 6 2 9 5 1 8 4
8 0 8 5 2 10 7 4 1 9 6 3
9 0 9 7 5 3 1 10 8 6 4 2
10 0 10 9 8 7 6 5 4 3 2 1
81 Solution We have
1 2+2 33 6
=
2+2 3+3 6 ( 2 + 2 3)2 (3 6)2
2+2 33 6
=
2 + 12 +
4 6 54
2+2 33 6
=
40 +4 6
( 2 + 2 3 3 6)(40 4 6)
=
2 (4 6)2
40
( 2 + 2 3 3 6)(40 4 6)
=
1504
16 2 + 22 3 30 6 18
=
376
82 Solution Since
(a)b1 = (ab1 ).
Similarly, from
a(b1 ) = (ab1 ).
h(a) = h(b) = a3 = b3
= a3 b3 = 0
= (a b)(a2 + ab + b2 ) = 0
Now,
b2 + ab + a2 = b +
a
2
2
+
3a2
4
.
This shews that b2 +ab+a2 is positive unless both a and b are zero. Hence
a b = 0 in all cases. We have shewn that h(b) = h(a) = a = b, and
the function is thus injective.
234 Appendix A
94 Solution We have
6a 6b
f(a) = f(b) =
2a 3 2b 3
6a(2b 3) = 6b(2a 3)
18a = 18b
a = b,
the results we deduce that f is bijective.
1 1 1
104 Answer A = 2 8
4
.
3 9 27
1 2 3
.
105 Answer A = 2 6
4
3 6 9
a + 1 0 2c
, 2a 4a 2c
.
106 Answer M + N = a 0 2M = 2b
b 2a
0 2a
2a 0 2 2a + 2b 0 2
107 Answer x = 1 and y = 4.
13
108 Answer A =
1
, B = 5 0
15 3 6 1
Some Answers and Hints 235
111 Solution The set of border elements is the union of two rows and two
columns. Thus we may choose at most four elements from the border,
and at least one from the central 3 3 matrix. The largest element of this
3 3 matrix is 15, so any allowable choice of does not exceed 15. The
choice 25, 15, 18,l 23, 20 shews that the largest minimum is indeed 15.
120 Answer
2 2
0 2
121 Answer
a b c a + b + c b+c c
AB = c + a b+c b
a+b
, BA = a + b + c a+b
a+b+c a+b+c a+b+c a+b+c c+a a
0 2 0 3
0 2 0 3
x 4 x 4 x 4 0 16 x2
and so we must have 16 x2 = 1 or x = 17.
and B = 0 1 . Then AB =
124 Solution Disprove! Take A =
1 0
0 0 0 0
B, but BA = 02 .
125 Solution Disprove! Take for example A =
0 0 and B = 1 0.
1 1 1 0
Then
A2 B2
1 06= 1 0.(A + B)(A B).
=
0 1 2 1
236 Appendix A
127 Answer
32 32
.
32 32
128 Answer A2003
=
0 21001 1002
3
.
21002 31001 0
130 Solution The assertion is clearly true for n = 1. Assume that is it true
for n, that is, assume
An
cos(n)
=
sin(n)
.
sin(n) cos(n)
Then
An+1 =
AAn
=
cos sin cos(n) sin(n)
sin cos sin(n) cos(n)
=
cos cos(n) sin sin(n) cos sin(n) sin cos(n)
sin cos(n) + cos sin(n) sin sin(n) + cos cos(n)
=
cos(n + 1) sin(n + 1),
sin(n + 1) cos(n + 1)
0 0 0
An In n1
" #I
n n2 2
= 3 + nI3 J+ 2 3 J
1 0 0 n n 0 #
"
+ 0 0 + 0
n
0 0
0 0 n 0
2
=
1
0
0 0 1 0 0 0 0 0 0
1 n n(n+1)
0 n
2
=
1
,
0 0 1
A2 B = A(AB) = AB = B
A3 B = A(A2 B) = A(AB) = AB = B
..
.
Am B = AB = B.
Hence B = Am B = 0n B = 0n .
a b. Using 135, deduce by iteration that
136 Hint Put A =
c d
Ak = (a + d)k1 A.
238 Appendix A
137 Answer
a , bc = a
b 2
c a
2
a b , a = 1 bc
138 Answer I , 2
c a
a b= X I. Then
139 Solution We complete squares by putting Y =
c d
a + bc b(a + d)= Y = X 2X+I = (XI) = 1 0+I = 0 0.
2
2 2 2
2
c(a + d) bc + d 6 3 6 4
tr (AC) + tr (DB) = n.
a contradiction, since n 1.
152 Solution Disprove! This is not generally true. Take A =
1 1 and
1 2
Some Answers and Hints 239
3 0. Clearly A
B= T
= A and BT = B. We have
0 1
3 1
AB =
3 2
but
(AB)T
3 3.
=
1 2
154 Solution We have
2 a b a b = tr
a + bc ab + bd = a +d +2bc
tr A = tr
2
2 2
c d c d ca + cd d + cb 2
and
tr
a b = (a + d) .
2
c d
Thus
tr A2 = (tr (A))2 a2 + d2 + 2bc = (a + d)2 bc = ad,
tr (A I4 )2 =
tr A2 2A + I4
= tr A 2tr (A) + tr (I )
2
4
= 4 2tr (A) + 4
= 2tr (A) ,
n
X
0 = cii = x2ik = xik = 0.
k=1
1 0 1 0
0 0 1 0
0 1 0 0 .
= P=
1 0 1 0
A1
1 1
so take
1 0
1 1
0 0
1 1 1 1 0 0 0 1
188 Solution Here is one possible approach.
a
d e f
b c g h i
d f
P:3 1
e
g h i
a b c
e d f
h g i
P 0 :C1 C2
b a c
h g g i
T :C1 C2 C1
e d d f
ba a c
h g g i
D:23 3
e d d f
2b 2a 2a 2c
Thus we take
0 0 1 0 1 0
P= 0 0
, P = 1 0 0,
0
1
1 0 0 0 0 1
1 0 0
1 0 0
T= 1 0
1
, D = 0 1 0.
0 0 1 0 0 2
where the entries appear on the j-column. Then we see that tr (AEij ) =
aji and similarly, by considering BEij , we see that tr (BEij ) = bji . Therefore
i, j, aji = bji , which implies that A = B.
.
..
. ...
0 0 ... 0
.. ..
. ... .
0 0 ... 0
216 Answer a = 1, b = 2.
(In + A)(In A + A2 A3 ) = In A + A2 A3 A + A2 A3 + A4 = In ,
A = AIn = (ABC)(BC)1 = 0n ,
0n = 0n C1 B1 A1 = (ABC)C1 B1 A1 = In ,
also gives a contradiction.
= AB = BA,
by cancelling A on the left and B on the right. One can also argue that
A = A1 , B = B1 , and so AB = (AB)1 = B1 A1 = BA.
225 Hint Observe that A = (a b)In + bU, where U is the n n matrix with
1F s everywhere. Prove that
227 Solution By Theorem 141 we have tr SAS1 = tr S1 SA = tr (A).
243 Answer The rank is 2.
244 Solution If B is invertible, then rank (AB) = rank (A) = rank (BA). Simi-
1 0
larly, if A is invertible rank (AB) = rank (B) = rank (BA). Now, take A =
0 0
and B =
0 1 . Then AB = B, and so rank (AB) = 1. But BA = 0 0 ,
0 0 0 0
and so rank (BA) = 0.
0 0 a2 b2 2(a b)
Performing R3 R4 R3 we have
1 a 1 b
0
1 a2 ba 1 ab
.
0 0 2a(a b) a(a b )
2
2
0 0 a2 b2 2(a b)
Factorising, this is
1 a 1 b
0
= .
1 a2 ba 1 ab
0
0 2a(a b) a(a b)(a + b)
0 0 0 a(a + 2 + b)(a b)(a 2 + b)
Some Answers and Hints 245
From R2 R3 we obtain
1 2 3 1 0 0
0 1
2 0 4 0
.
0 6 2 5 1 0
We deduce that
1
1 2 3 4 2 0
2 1 = 4
3
2 0
.
3 1 2 0 4 2
Performing R1 R3 , R3 R3 , in succession,
1 a b 0 0 1
.
0 0
1 a 0 1
0 0 1 1 0 0
whence
b a 2
a 1
= a 0
B1 1
.
1 0 0
Now,
0 0 1 a b c b a 2
a 1
0 1 a 1 0 0 a 0
BAB1 =
1
1 a b
0 1 0 1
0 0
1 a 0 a 1 0
2
0 1 0 b a a 1
=
0 0 c 1 0 0
a 1 0
b 0 1
=
c 0 0
= AT ,
B1 2
a2 5+2a
a+4
a2 5+2a
1
a2 5+2a
.
2a
a2 5+2a
a22a+5
5+2a
a
a2 5+2a
Thus B is invertible whenever a 6= 1 6.
248 Appendix A
1 1 1
Perform R1 R1 , R2 R2 , R3 R3 , in succession, obtaining
a b a
1 2 3 1/a 0 0
0 0
1 2 0 1/b
.
0 0 1 0 0 1/c
Whence 1
a 2a 3a 1/a 2/b 1/c
0 2b = 2/c
b
0 1/b
.
0 0 c 0 0 1/c
1 b 1
0 2a 2ac 2c
c 1 1
0 c b 2ba 2a 2b
as long as abc 6= 0.
Perform 1
ab+bc+ca+abc R1 R1 . The matrix turns into
1 1 1 bc ca ab
ca
ab+bc+ca+abc ab+bc+ca+abc ab+bc+ca+abc
ca + abc ca 0 ca 0
.
ab ab ab + abc 0 0 ab
.
2
abc c2 a2 2
a bc
abc 0 ab+bc+ca+abc ca ab+bc+ca+abc ab+bc+ca+abc
2 2
ab c a bc a 2 b2
0 0 abc ab+bc+ca+abc ab+bc+ca+abc ab ab+bc+ca+abc
Perform 1
ABC R2 R2 and 1
ABC R3 R3 . We obtain
1 1 1 bc ca ab
0
ab+bc+ca+abc ab+bc+ca+abc ab+bc+ca+abc
.
c 1 ca a
1 0 ab+bc+ca+abc b b(ab+bc+ca+abc) ab+bc+ca+abc
b a 1 ab
0 0 1 ab+bc+ca+abc ab+bc+ca+abc c c(ab+bc+ca+abc)
.
c 1 ca a
1 0 ab+bc+ca+abc b b(ab+bc+ca+abc) ab+bc+ca+abc
b a 1 ab
0 0 1 ab+bc+ca+abc ab+bc+ca+abc c c(ab+bc+ca+abc)
Some Answers and Hints 251
c c+a+ca a
ab+bc+ca+abc ab+bc+ca+abc ab+bc+ca+abc
b a a+b+ab
ab+bc+ca+abc ab+bc+ca+abc ab+bc+ca+abc
271 Solution Since rank A2 < 5, A2 is not invertible. But then A is not
invertible and hence rank (A) < 5.
2y + w = 2 = 2y = 2 w = y = 1 + w,
and
x + y + z + w = 0 = x = y z w = 2y + 2z + 2w.
Hence
x 0 0 0
y 1 0 0
= + z + w .
z 0 1 0
w 0 0 1
Hence
1
x 1 2 3 5 4 2 0 5 4
y = 2 1 6 = 2 4
3
0
6= 3.
z 3 1 2 0 0 4 2 0 3
291 Solution Observe that the third row is the sum of the first two rows
252 Appendix A
and the fourth row is twice the third. So we have
1 1 1 1 1 1
1 1 1 1 1 1
1 1
1 1
0 1 0 1
0 1 0 1
2 0
R3 R1 R2 R3
0 0
4 1 2 1 2
0
R4 2R1 2R2 R4
0 0 0 0 0
0
2 4 2 4
0 0 0 0
1 0 0 0 1 0
1 0 0 0 1 0
0 1 1 1 0 1
0 1
0 1 0 0
R2 R5 R2
0 0
R1 R5 R1
0 0 0 0 0
0
0 0 0 0
1 0 0 0 1 0
0
0 0 0 0
0 0 0 0 0 0
1
1
292 Answer The unique solution is 1 .
1
1
Performing R1 R2 .
1 2m 1 4m
.
2m 2
1 1
1 1 2m 2m2
Performing R2 R3 .
1 2m 1
4m
1 2m
.
2
1 2m
2m 1 1 2
0 1 4m2 1 2m 2 8m2
If m = 1
the matrix becomes
2
1 1 1 2
0
0 0 23
0 0 0 0
254 Appendix A
0 1 + 2m 1 2(1 + 2m)
.
13m
12m
(1+2m)(1m)
z 12m
ax + y 2z = 1,
(a 2)z = 0.
ax + y = 1,
(a 1)2 y = 1 a,
2x + (3 a)y = 1.
x + y = 1,
2x + 2y = 1,
.
1
a1
z 0
295 Answer The system is solvable if m 6= 0, m 6= 2. If m 6= 2 there is the
x 1
solution y=
m2
.
m+3
m2
m+2
z m2
y c d b + a
x a+d+bc
x b a 0 c
y c 0 a b
=
z 0 c b a
1 1
a
c
2b 2c 2bc
=
b ,
1 b 1
2a 2ac 2c
c 1 1
a
b + c a
2ba 2a 2b
2bc
2 2 2
a + c b
a +2ac
2 2 2
b c
=
2 2 2
2ab
as long as the inverse matrix exists, which is as long as abc 6= 0
298 Answer
x = 22 36
y = 23 312
z = 22 37 .
a1 = b1 ,
a2 = b2 ,
a3 = b3 ,
Some Answers and Hints 257
a4 = b4 ,
a1 b2 = 3,
a1 b3 = 6,
a1 b4 = 9.
0
0 1 y 1
1 0 0 1 y 0
0
y 1 0 0
y 1 0 0 1 0
0
y 1 1 y
0 1 0 y 1 y2 0
258 Appendix A
0
0 1 y2 y1 y
0 0 y y 1 1 y2 0
0
0 0 y3 + 2y 1 y2 + y 1
0 0 0 y2 + y 1 1 y y2 0
Performing R5 + R4 R5 we get
1 0 0 1 y 0
0 0
1 y 1 0
.
0 0
0 0 1 y 1
0
0 0 y3 + 2y 1 y2 + y 1
0 0 0 y3 + y2 + 3y 2 0 0
0
0 0 (y 1)(y2 + y 1) y2 + y 1
0 0 0 (y 2)(y2 + y 1) 0 0
Some Answers and Hints 259
0
0 0 5 5
0 0 0 0 0 0
2
4
x5 t
0
0 0 0 0
0 0 0 0 0 0
2
x = ys t ,
2
(s, t) R2 .
3
x s
4
x5 t
260 Appendix A
Since y2 s s = (y2 + y 1)s ys, this last solution can be also written as
x ys yt
1 x yt s
x = ys t ,
2
(s, t) R2 .
x s
3
4
x5 t
Finally, if (y 2)(y2 + y 1) 6= 0, then x4 = 0, and we obtain
x 0
x 0
1
2
331 Solution We have 2BC = BE + EC. By Chasles Rule AC = AE + EC,
and BD = BE + ED. We deduce that
AC + BD = AE + EC + BE + ED = AD + BC.
But since ABCD is a parallelogram, AD = BC. Hence
AC + BD = AD + BC = 2BC.
332 Solution We have IA = 3IB IA = 3(IA + AB) = 3IA 3AB.
Thus we deduce
IA + 3IA = 3AB 4IA = 3AB
4AI = 3AB
AI = 43 AB.
Similarly
JA = 31 JB 3JA = JB
3JA = JA AB
4JA = AB
AJ = 41 AB
.
Thus we take I such that AI = 43 AB and J such that AJ = 41 AB.
Now
MA + 3MB = MI + IA + 3IB
= 4MI + IA + 3IB
= 4MI,
and
3MA + MB = 3MJ + 3JA + MJ + JB
= 4MJ + 3JA + JB
= 4MJ.
X+Y O + P R(O P)
= +
2 2 2
is independent of the position of the gallows. This gives a simple algo-
rithm for treasure-finding: take P as the (hitherto) arbitrary origin, then
O + R(O)
the treasure is at .
2
352 Answer a = 1
2
354 Answer
4 1 2
p = = 2 + 3 = 2r + 3s.
5 1 1
a = (ai)i + (aj)j
356 Solution
a + b = 0 = a(a + b) = a0
= (aa) = 0
2
= ||a|| = 0.
a + b = 0 = b = 0
= = 0,
since b 6= 0.
But
9
(2x + 3y)(2x 3y) = 4||x||2 9||y||2 = 4( ||y||2 ) 9||y||2 = 0.
4
Some Answers and Hints 263
= aa 2ab + bb
2 2
= ||a|| 2ab + ||b|| ,
= 4uv,
projax projax a
proj = 2
a
||a||
a
ax
||x||2
xa
= 2
a
||a||
(ax)2
= 2 2
a,
||x|| ||a||
(ax)2
Since 0 2 2
1 by the CBS Inequality, the result follows.
||x|| ||a||
362 Solution Clearly, if a = 0 and 6= 0 then there are no solutions. If both
a = 0 and = 0, then the solution set is the whole space R2 . So assume
that a 6= 0. By Theorem 347, we may write x = u + v with projxa = u||a and
v a. Thus there are infinitely many solutions, each of the form
x a
x=u+v= 2
a+v= 2
a + v,
||a|| ||a||
where v a .
264 Appendix A
372 Solution Since a =
is normal to 2x y = 1 and b = 1 is
2
1 3
normal to x 3y = 1, the desired angle can be obtained by finding the
angle between the normal vectors:
(a, b) = arccos
ab
||a||||b||
5 1
= arccos = arccos = .
5 10 2 4
373 Answer 2(x 1) + (y + 1) = 0 or 2x + y = 1.
374 Solution By Chasles Rule AA 0 = AG + GA 0 , BB 0 = BG + GB 0 , and
0 0
CC = CG + GC . Thus
0 = AA 0 + BB 0 + CC 0
= AG + GA 0 + BG + GB 0 + CG + GC 0
= (GA + GB + GC) + (GA 0 + GB 0 + GC 0 )
= GA 0 + GB 0 + GC 0 ,
EB EI = l(ED EJ 0 ) = IB = lJ 0 D.
Since I is the midpoint of [A, B], IA+ IB = 0, and thus l(J 0 C+ J 0 D) = 0.
Since l 6= 0, we deduce J 0 C + J 0 D = 0, that is, J 0 is the midpoint of
[C, D], and so J 0 = J.
1
AE = 4 AC AB + BE = 14 AC ,
and
3
AF = 43 AC AD + DF = 4 AC .
Adding, and observing that since ABCD is a parallelogram, AB =
CD,
AB + BE + AD + DF = AC BE + DF = AC AB AD
BE + DF = AD + DC AB AD .
BE = DF.
The last equality shews that the lines (BE) and (DF) are parallel.
Observe that BJ = 21 BC = 21 AD = AI = IA . Hence
IJ = IA + AB + BJ = AB,
Observe that
1 1 1
IE = IA + AE = DA + AC = CB + FC = CJ + FC = FC + CJ = FJ,
2 4 2
whence IEJF is a parallelogram.
377 Solution Since IE = 31 ID and [I, D] is a median of 4ABD, E is the
centre of gravity of 4ABD. Let M be the midpoint of [B, D], and observe
that M is the centre of the parallelogram, and so 2AM = AB + AD. Thus
2 1 1
AE = AM = (2AM) = (AB + AD).
3 3 3
To shew that A, C, E are collinear it is enough to notice that AE = 13 AC.
266 Appendix A
||[A, B]||
378 Solution Suppose A, B, C are collinear and that = . Then
||[B, C]||
by the Section Formula 4.4,
c + a
b= ,
+
whence a ( + )b + c = 0 and clearly ( + ) + = 0. Thus we
may take = , = + , and = . Conversely, suppose that
a + b + c = 0, ++=0
for some real numbers , , , not all zero. Assume without loss of gener-
ality that 6= 0. Otherwise we simply change the roles of , and and
Then = ( + ) 6= 0. Hence
a + b
a + b = ( + )c = c = ,
+
and thus [O, C] divides [A, B] into the ratio , and therefore, A, B, C are
collinear.
379 Solution Put OX = x for X {A, A 0 , B, B 0 , C, C 0 , L, M, N, V}. Using prob-
lem 378 we deduce
v + a + 0 a 0 = 0, 1 + + 0 = 0, (A.1)
v + a + 0 a 0 = 0, 1 + + 0 = 0, (A.2)
v + a + 0 a 0 = 0, 1 + + 0 = 0. (A.3)
From A.2, A.3, and the Section Formula 4.4 we find
b c 0b 0 0c 0
= = l,
0 0
whence ( )l = b c. In a similar fashion, we deduce
( )m = c a,
( )n = a b.
This gives
( )l + ( )m + ( )n = 0,
( ) + ( ) + ( ) = 0,
and appealing to problem 378 once again, we deduce that L, M, N are
collinear.
Some Answers and Hints 267
391 Answer [A] AS, [B] AB.
Then either
3
= =
2
3a
||a||
3a
2 3
2 ,
0
or
3
=
2
3a
||a|| 3
2
0
397 Solution
398 Answer
(ab)a + 6b + 2ac
x= 2
12 + 2||a||
(a c)a + 6c + 3ab
y= 2
18 + 3||a||
399 Solution Assume contrariwise that a, b, c are three unit vectors in R3
2 1
such that the angle between any two of them is > . Then ab < ,
3 2
1 1
bc < , and ca < . Thus
2 2
2 2 2 2
||a + b + c|| = ||a|| + ||b|| + ||c||
< 1+1+1111
= 0,
and
0 (a) a
1 1 = 0
2a 0 2a
are coplanar. A vector normal to the plane is
a 2a a
The equation of the plane is thus given by
2a x a
3a y 0 = 0,
2
a za
Some Answers and Hints 269
that is,
2ax + 3a2 y az = a2 .
411 Solution The vectorial form of the equation of the line is
1 1
r= 0+ t 2.
1 1
1 1
Since the line follows the direction of 2, this means that 2is nor-
1 1
mal to the plane, and thus the equation of the desired plane is
(x 1) 2(y 1) (z 1) = 0.
412 Solution Observe that (0, 0, 0) (as 0 = 2(0) = 3(0)) is on the line, and
hence on the plane. Thus the vector
1 0 1
1 0 = 1
1 0 1
Hence, the vectorial form of the equation of the line is
0 1 1
r= 0+ t 1/2= t 1/2.
0 1/3 1/3
1
This means that 1/2also lies on the plane, and thus
1/3
1 1 1/6
1 1/2 = 4/3
1 1/3 3/2
270 Appendix A
1 4 3
x y + z = 0.
6 3 2
z 1/c
1/a
1/b is perpendicular to the plane.
Thus the vector
Therefore, the
1/c
equation of the plane is
1/a x 1 0
1/b y 1 = 0 ,
1/c z1 0
or
x y z 1 1 1
+ + = + + .
a b c a b c
We may also write this as
a2
the same direction as this vector, thus the equation of the line is
x 0 a
y = 0 + t a ,
t R.
2
z 1 a2
Some Answers and Hints 271
x z y = 1 = 1 y = 1 = y = 2.
Hence if z = t,
x t 1 1 1
y = 2 = 2 + t 0 .
z t 0 1
1 z+1
bc = ab ca = 2k + 3i + i 2j = 4i 2j 2k.
418 Answer 4x + 6y = 1
419 Answer There are 7 vertices (V0 = (0, 0, 0), V1 = (11, 0, 0), V2 = (0, 9, 0), V3 =
(0, 0, 8), V4 = (0, 3, 8), V5 = (9, 0, 2), V6 = (4, 7, 0)) and 11 edges (V0 V1 , V0 V2 ,
V0 V3 , V1 V5 , V1 V6 , V2 V4 , V3 V4 , V3 V5 , V4 V5 , and V4 V6 ).
Pn
428 Solution Observe that k=1 1 = n. Then we have
1 = ! X a X a1 ,
n 2 n 2 n n
X X 1
2 2
n = (ak ) k 2
ak k
k=1 k=1 k=1 k=1
429 Solution This follows at once from the CBS Inequality by putting
a1
1
2
1
v = , u=
a2
. . . . . .
2
an
n n
443 Solution We expand (1F + 1F )(a + b) in two ways, first using 5.7 first
and then 5.8, obtaining
(1F + 1F )(a + b) = 1F (a + b) + 1F (a + b) = a + b + a + b.
a + a + b + b = a + b + a + b.
a + b = b + a,
444 Solution We must prove that each of the axioms of a vector space
are satisfied. Clearly if (x, y, ) R+ R+ R then xy = xy > 0 and x =
x > 0, so V is closed under vector addition and scalar multiplication.
Commutativity and associativity of vector addition are obvious.
x A = x = xA = x = A = 1.
Now
(x y) = (xy) = x y = x y = ( x) ( y),
and
( + ) x = x+ = x x = (x ) (x ) = ( x) ( x),
Finally,
1 x = x1 = x,
and
( x) = ( x) = (x ) = x = () x,
and the last two axioms also hold.
445 Answer C is a vector space over R, the proof is trivial. But R is not
a vector space over C, since, for example taking i as a scalar (from C)
and 1 as a vector (from R) the scalar multiple i 1 = i 6 R and so there is
no closure under scalar multiplication.
1 0 1 0 1 0 1
b b
x = X, y = X,
0
c a b 3d = 0,
c a 0 b 0 3d 0 = 0.
0
d d0
Then
a a a + a
0 0
0 0
d d0 d + d 0
Observe that
(a + a 0 ) (b + b 0 ) 3(d + d 0 ) = (a b 3d) + (a 0 b 0 3d 0 ) = 0 + 0 = 0,
1
1
2 2
u = 5b , v = 5b ,
a + 2b a + 2b
1 2 R.
1
1
2 2
a1 a2
Some Answers and Hints 275
Put s = a1 + a2 , t = b1 + b2 . Then
a + a s
2(a + a ) 3(b + b ) 2s 3t
1 2
1 2 1
2
u + v = = 5t X,
(a + a ) + 2(b + b ) s + 2t
5(b + b1 ) 2
1 2 1
2
a1 + a2 s
since this last matrix has the basic shape of matrices in X. This shews that
X is a vector subspace of R5 .
a(u + v) = au + av = 0 + 0 = 0,
a(u + v) = au + av = 0 + 0 = 0,
462 Solution We shew that some of the properties in the definition of vec-
tor subspace fail to hold in these sets.
0 0
Take x = 1, = 2. Then x V but 2x = 26 V as 0 2
+ 22 = 4 6= 1.
0 0
So V is not closed under scalar multiplication.
0 1 1
Take x = 1, y = 0. Then x W, y W but x + y = 1 6 W as
0 0 0
1 1 = 1 6= 0. Hence W is not closed under vector addition.
Take x =
1 1. Then x Z but x = 1 1= 1 1
6 Z
0 0 0 0 0 0
as 1 + (1) = 2 6= 0. So Z is not closed under scalar multiplication.
2
276 Appendix A
a + a b = b X,
b + a b = a (V \ X) {0},
a contradiction since a X.
. . .
465 Solution Assume contrariwise that V = U1 U2 Uk is the short-
est such list. Since the Uj are proper subspaces, k > 1. Choose x
/ . .
U1 , x 6 U2 Uk and choose y 6 U1 . Put L = {y + x| F}. Claim:
/
L U1 = . For if u L U1 then a0 F with u = y + a0 x and so
y = u a0 x U1 , a contradiction. So L and U1 are disjoint.
It is easy to verify that these subspaces satisfy the conditions of the prob-
lem.
Some Answers and Hints 277
476 Solution If
1 1 1
a 0+ b 1+ c 1= 0,
0 0 1
then
a + b + c 0
b + c = 0 .
c 0
Then
a + b + c + d = 0,
a + b c + d = 0,
a b + c = 0,
a b c + d = 0.
Subtracting the second equation from the first, we deduce 2c = 0, that
is, c = 0. Subtracting the third equation from the fourth, we deduce
2c + d = 0 or d = 0. From the first and third equations, we then deduce
a + b = 0 and a b = 0, which entails a = b = 0. In conclusion, a = b =
c = d = 0.
Now, put
1 1 1 1 1
1 1 1 1 2
x + y + z + w = .
1 1 1 0 1
1 1 1 1 1
278 Appendix A
Then
x + y + z + w = 1,
x + y z + w = 2,
x y + z = 1,
x y z + w = 1.
Solving as before, we find
1 1 1 1 1
1 1 1 1 1 1 2
2 + = .
1 2 1 2 1 0 1
1 1 1 1 1
a + b + ab = 0. (A.4)
2
Since a(ab) = 0, taking the dot product of A.4 with a yields ||a|| = 0,
which means that = 0, since ||a|| 6= 0. Similarly, we take the dot product
with b and ab obtaining respectively, = 0 and = 0. This establishes
linear independence.
1 a1 + + k ak = 0.
Taking the dot product with aj and using the fact that ai aj = 0 for i 6= j
we obtain
2
0 = 0aj = j aj aj = j ||aj || .
2
Since aj 6= 0 = ||aj || 6= 0, we must have j = 0. Thus the only lin-
ear combination giving the zero vector is the trivial linear combination,
which proves that the vectors are linearly independent.
484 Solution Yes. Suppose that a + b 2 = 0 is a non-trivial linear combi-
nation of 1 and 2 with rational numbers a and b. If one of a, b is different
from 0 then so is the other. Hence
b
a + b 2 = 0 = 2 = .
a
b
The sinistral side of the equality 2= is irrational whereas the dextral
a
side is rational, a contradiction.
485 Solution No. The representation
2 1 + ( 2) 2 = 0 is a non-trivial
linear combination of 1 and 2.
If ac 6= 0, then
2b2 a2 3c2
b 2 = a c 3 2b2 = a2 + 2ac 3 + 3c2 = 3.
2ac
The dextral side of the last implication is irrational, whereas the sinis-
tral side is rational. Thus it must be the case that ac = 0. If a = 0, c 6= 0
then
b 2+c 3=0 =
b
c
3
2
0,
and again the dextral side is irrational and the sinistral side is ratio-
nal. Thus if a = 0 then also c = 0. We can similarly prove that c = 0
entails a = 0. Thus we have
b 2 = 0,
2. Rationalising denominators,
1 2 1 + 2 2 12 + 4
+ = +
1 2 12 2 12 12 4
1 1
= 1 2 + 3+
2 2
1 1
= 2+ 3.
2 2
280 Appendix A
Then
c = ae2x bex .
Letting x +, we obtain c = 0. Thus
aex + be2x = 0,
and so
b = aex .
Again, letting x +, we obtain b = 0. This yields
aex = 0.
which implies
cos 2x cos2 x + sin2 x = 0.
s = p(1) = a b + c d R.
Now,
p 0 (x) = b + 2cx + 3dx2 = t + 2u(1 + x) + 3v(1 + x)2 .
Letting x = 1 we find
t = p 0 (1) = b 2c + 3d R.
Again,
p 00 (x) = 2c + 6dx = 2u + 6v(1 + x).
Some Answers and Hints 281
Letting x = 1 we find
u = p 00 (1) = c 3d R.
Finally,
p 000 (x) = 6d = 6v,
b = 1,
a b = 1,
1 1 0
which is impossible. Thus 1 is not a linear combination of 0 , 1
1
1 1
1 0
0 , 1 .
and hence is not in span
1 1
503 Solution It is
1 0+ b 0 0+ c 0 1= a c,
a
0 0 0 1 1 0 c b
i.e., this family spans the set of all skew-symmetric 2 2 matrices over R.
282 Appendix A
0 , 5
1 2
1 0
a + f = 0,
a+b=0
b+c=0
c+d=0
d + f = 0.
Solving we find a = b = c = d = f = 0, which means that the
{v1 + v2 , v2 + v3 , v3 + v4 , v4 + v5 , v5 + v1 }
(The second 1 occurs on the n-th position. The 1s migrate from the
2nd and n+1-th position on a1 to the n1-th and 2n-th position on an1 .)
284 Appendix A
b b
u = , v = ,
0
c b + 2c = 0,
c b 0 + 2c 0 = 0.
0
d d0
We have
a + a 0
b + b
u + v =
c + c ,
0
0
d + d 0
and to demonstrate that u + v X we need to shew that (b + b 0 ) +
2(c + c 0 ) = 0. But this is easy, as
(b + b 0 ) + 2(c + c 0 ) = (b + 2c) + (b 0 + 2c 0 ) = 0 + 0 = 0.
Now
a a 1 0 0
b 2c 0 2 0
= = a + c + d
c c 0 1 0
d d 0 0 1
It is clear that
1 0 0
0 2 0
, ,
0 1 0
0 0 1
are linearly independent and span X. They thus constitute a basis for X.
n(n + 1)
520 Answer As a basis we may take the matrices Eij Mn (F)
2
for 1 i j n.
x = a + b + ab.
We then have
b = ax
= ab + a(ab)
= ab + ((ab)a (aa)b).
= ab (aab)
2
= ab ||a|| b,
from where
2
ab + (||a|| 1)b = 0,
which means that = 0 and = ||a|| 2 , since a, b, ab are linearly
1
independent. Thus
1
x = a 2
ab
||a||
in this last case.
1 1 1
1
1 1 = 4I ,
=
1 1
A2
1 1
4
1 1
1 1 1 1
286 Appendix A
and so
x 1 1/4 1/4 1/4 1/4
1 5/4
y 2 1/4 1/4 2 = 1/4 .
= A1
= 1/4 1/4
z 1 1/4 1/4 1/4 1/4 1 1/4
w 1 1/4 1/4 1/4 1/4 1 1/4
It follows that
1 1 1 1 1
2 5 1 1 1 1 1 1 1
= 4 + 4 4 4 .
1 1 1 1 1
1 1 1 1 1
5 1 1 1
, , ,
4 4 4 4
. !
Some Answers and Hints 287
3. Since we have
1 1 1 1 1
2 5 1 1 1 1 1 1 1
= 4 4 + 4 4 ,
1 1 1 1 1
1 1 1 1 1
5 1 1 1
, , ,
4 4 4 4
. !
1 0 0 0 1 0
A= 1 0 B= 0
1
, 1 1
.
1 1 1 1 1 1
1
1 0 0
0 1 0 1
= 1 0 0 0
1 1 0 0
= 1 1 0
.
B= 1 0 1 0 0 0 0
1
P=A 1 1 1 1 0
1 1 1 1 1 1 0 1 1 1 1 1 0 0 1
Also
0 1 0 1 2
2 = 1 .
1 0
0
0 0 1 0 0
288 Appendix A
541 Solution Let R. Then
x + a (x + a) (y + b) (z + c)
L y + b (x + a) + (y + b) + (z + c)
=
z + c z + c
x + y + z + a + b + c
x y z a b c
=
z c
x a
= L y+ L b,
z c
L((x, y) + (x 0 , y 0 )) = L(x + x 0 , y + y 0 )
= (x + x 0 )k + h(y + y 0 )
= xk + x 0 k + hy + hy 0
= L(x, y) + L(x 0 , y 0 )
543 Solution
L(H + H 0 ) = A1 (H + H 0 )A1
= A1 HA1 + (A1 H 0 A1 )
= L(H) + L(H 0 ),
that is,
(1 )a + b T (S),
as we wished to show.
x
551 Solution Assume y ker (L). Then
z
x 0
L y= 0,
z 0
that is
x y z = 0,
x + y + z = 0,
z = 0.
This implies that x y = 0 and x + y = 0, and so x = y = z = 0. This means
that
0
ker (L) =
0
0
,
and L is injective.
By the Dimension Theorem 548, dim Im (L) = dim V dim ker (L) =
3 0 = 3, which means that
Im (L) = R3
and L is surjective.
a
552 Solution Assume that b ker (T ),
c
a 1 1 0
b = (a b) 0 + b 1 + c 0 .
c 0 0 1
290 Appendix A
Then
0 a
0
T b
0
=
c
0
1 1 0
= (a b)T 0+ bT 1+ cT 0
0 0 1
1 2 1
0 1 1
(a b) + b + c
=
1 0 1
0 0 0
a + b + c
b c
.
a + b + c
=
0
ker (T ) = c 1 : c R ,
1
and so
)
'' 1 1 **
'' 0 1 **
Im (T ) = span ' &' , **.
'(1 1 +*
0 0
Then x = 2y and so
x= y 2.
y 1
This means that dim ker (L) = 1 and ker (L) is the line through the origin
and (2, 1). Observe that L is not injective.
By the Dimension Theorem 548, dim Im (L) = dim V dim ker (L) =
a
2 1 = 1. Assume that b Im (L). Then (x, y) R 2
such that
and so L is injective.
By the Dimension Theorem 548, dim Im (L) = dim V dim ker (L) =
a
2 0 = 2. Assume that b Im (L). Then (x, y) R 2
such that
x x y z a
Now, if
L y= y 2z = b .
z
Then
a= x y z= x 1+ y 1+ z 1.
b y 2z 0 1 2
Now,
1 1 1
3 2 =
0 1 2
and
1, 1
0 1
Then a = d and so,
a b= d b= d 1 0+ b 0 1+ c 0 0,
c d c d 0 1 0 0 1 0
and so dim ker (L) = 3. Thus L is not injective. L is surjective, however. For
if R, then
0 .
= tr
0 0
L(A + B) = (A + B)T + (A + B)
= AT + BT + A + B
= AT + A + BT + B
= L(A) + L(B),
proving that L is linear.
3. By the Dimension Theorem, dim Im (L) = 4 1 = 3. As above,
L(A) =
2a b + c
2d
b+c
= a
2 0+ (b + c) 0 1+ d 0 0,
0 0 1 0 0 2
Some Answers and Hints 295
from where
2 0, 0 1, 0 0 .
Im (L) = span
0 0 1 0 0 2
1 + 2 + c = 0 = c = 1.
1
2. Observe that 1 ker (T ) and so
1
1 0
T 1= 0.
1 0
Thus
1 2 1 3
T 0= T 1 T 1= 2 ,
0 1 1 5
0 1 1 1
T 1= T 2 T 1= 2 ,
0 1 1 1
0 1 1 1
T 0= T 1 T 1= 0 .
1 2 1 1
The required matrix is therefore
3 1 1
.
2 0
2
5 1 1
296 Appendix A
x + u + y + v
x + u y v
=
2(x + u) + 3(y + v)
x + y u + v
x y + u v
=
2x + 3y
2u + 3v
x
T
+ T
,
u
y v
=
2. We have
0
x + y 0
x 0
T
= x y = 0 x = y = 0,
y
0 2x + 3y 0
x + y 1 1
x x y = x 1 + y 1 ,
T
=
y
2x + 3y 2 3
whence
1 1
1 , 1
Im (T ) = span
.
2 3
Some Answers and Hints 297
4. We have
8 1 1 0 13/2
B
and
11 1 1 0 19/2
B
13/2 19/2
B
564 Solution The matrix will be a 2 3 matrix. In each case, we find the
action of L on the basis elements of R3 and express the result in the given
basis for R3 .
1. We have
1 1 0 2 0 0
0 = ,L 1 = ,L 0 = .
L
3 0 1
0 0 1
2. We have
1 1 1 3 1 3
0 = ,L 1 = ,L 1 = .
L
3 3 2
0 0 1
298 Appendix A
3. We have
1 1 1 1 2
0 = = 2 + 3 = ,
L
3 0 1 3
A
0
1 3 1 1 0
1 = = 0 + 3 = ,
L
3 0 1 3
0 A
1 3 1 1 1
1 = = 1 + 2 = .
L
2 0 1
A
2
1
The required matrix is
2 0 1
.
3 3 2
2
565 Solution Observe that Im (T ) = ker (T ) and so
3
2 0
T = .
3 0
Now
1 1 2 1 2 6
T = T
3 = 3T T = ,
0 1 3 1 3 9
and
0 2 1 2 1 4
T = T
2 = T 2T = .
1 3 1 3 1 6
Some Answers and Hints 299
0 1 = 0,
tr
0 0
0 0 = 0,
tr
1 0
0 0 = 1.
tr
0 1
Thus
ML = (1 0 0 1).
567 Solution First observe that ker (B) ker (AB) since X Mq1 (R),
BX = 0 = (AB)X = A(BX) = 0.
Now
dim ker (B) = q dim Im (B)
= q rank (B)
= q rank (AB)
= q dim Im (AB)
Thus ker (B) = ker (AB) . Similarly, we can demonstrate that ker (ABC) =
300 Appendix A
= dim Im (BC)
= rank (BC) .
ond column by b, and its third column by c, we obtain
abc abc abc
abc = a
.
2
b2 c2
a3 b3 c3
We may factor out abc from the first row of this last matrix thereby ob-
taining
1 1 1
.
abc = abc det a c
2
b2 2
a3 b3 c3
Upon dividing by abc,
1 1 1
= det a c
.
2
b2 2
a3 b3 c3
622 Solution Performing R1 + R2 + R3 R1 we have
a b c 2a 2a
= det 2b
bca 2b
2c 2c cab
a + b + c a + b + c a + b + c
= det 2b
bca 2b
.
2c 2c cab
Some Answers and Hints 301
Performing C2 C1 C2 and C3 C1 C3 ,
1 0 0
= (a + b + c) det 2b
b c a 0
.
2c 0 c a b
as wanted.
(det A)3 27
(det A3 B3 C1 ) = 3
= .
(det B) (det C) 16
21 a22 0 0
X= a
. a32 a33 0
..
31
.. .. .. ..
. . . .
and
..
a12 a13 . a1n
0 ..
a23 . a2n
Y = 0
..
. 0 . a3n
..
.. .. .. ..
. . .
..
.
0 0 0 .
637 Solution Since the second column has three 0s, it is advantageous
to expand along it, and thus we are reduced to calculate
1 1 1
det 2 1
3+2
3(1) 0
1 0 1
Expanding this last determinant along the second column, the original
determinant is thus
3(1)3+2 (1)(1)1+2
2 1= 3(1)(1)(1)(1) = 3.
det
1 1
Some Answers and Hints 303
a
0 0
1 1 1
det b 0
0
=
x det 0 b 0
0 0 c
0 0 c
1 1 1 1 1 1
+x det a 0 0 x det a 0 0
0 0 c
0 b 0
1 1 1 1 1 1
xabc xbc + x det a 0 0 x det a 0 0
=
.
0 0 c 0 b 0
It follows that
abc = x(bc + ab + ca),
whence
1 bc + ab + ca 1 1 1
= = + + ,
x abc a b c
as wanted.
304 Appendix A
639 Solution Expanding along the first row the determinant equals
a b 0
a 0 0
a b+ ab det a b
a det 0 b b ab det
0
+ b det 0 a
=
1 1 1 1
1 1 1 1 1 1
= 2ab(a b),
as wanted.
640 Solution Expanding along the first row, the determinant equals
a 0 b 0 a b
a det 0 0 0
d
+ b det c 0
.
c 0 d 0 c d
Expanding the resulting two determinants along the second row, we ob-
tain
+b(c) det a b= ad(adbc)bc(adbc) = (adbc) ,
ad det
a b 2
c d c d
as wanted.
1 1= 1 = (1)
det 2+1
.
1 0
Assume that the result is true for n 1. Expanding the determinant along
Some Answers and Hints 305
1 1 1 1 1
0
..
0
1 ..
0 1
0 . 0
0
.
0 0 0
0 0
0 0
det 0 1 det 0 0
1 0 0
. 0
=
. 1 0
..
0 1 0 .. ..
..
.. .. .. . .
. . .
0 0 1 0
0 0 0 1 0
1 1 1 1
1 ..
0
0 .
1 det
0 0
0
1
. 0 0
..
.. ..
. .
0 0 1 0
= 1(0) (1)(1)n
= (1)n+1 ,
1 n 1 n 1 n 1 n1
n ..
2n 0 0 . 0
det A = det
n .
n
0 3n 0 0
. 0 0 4n 0
..
.. .. ..
. . .
n 0 0 0 0 0
306 Appendix A
0 0 0 1 0
. . 1 0 0
.. ..
.. ..
. .
0 0 0
1 0
1 1 1 . 1 1
1 0 0 .. 0
0
(n!) det
0 1 0 0
0 0 1
0
0
=
. . 0
.. .. ...
..
.
0 0 0 1 0
= (n!)(1)n
= (1)n+1 n!,
X ,n n
k-
n
=2
k=0
Some Answers and Hints 307
and
n
X
,n
k=0
(1) k
k - = 0, if n > 0.
"#
Assume that n is odd. Observe that then there are n + 1 (an even num-
" #
ber) of columns and that on the same row, n k is on a column of opposite
parity to that of nk
n
. By performing C1 C2 + C3 C4 + + Cn Cn+1
C1 , the first column becomes all 0s, whence the determinant if 0 if n is
odd.
If
1 1 a 0
=
0 0 b 0
then a = b. Thus
a= a 1
b 1
1
and we can take as the eigenvector corresponding to = 0. Simi-
1
larly, for = 2,
1 3,
2I A = 2
1 3
308 Appendix A
If
1 3 a= 0
0 0 b 0
1
and we can take as the eigenvector corresponding to = 2.
3
= (2 3 4) + 2(2 2) + ( 1)
= ( 4)( + 1) 5( + 1)
= (2 4 5)( + 1)
= ( + 1)2 ( 5)
If = 1,
1 2 1 a 0
A) = 2 2
(I3 4
b= 0 a = 2b + c
1 2 1 c 0
a 2 1
b = b 1 + c 0 .
c 0 1
2 1
We may take as eigenvectors 1 , 0, which are clearly linearly
0 1
independent.
If = 5,
5 2 1 a 0
A) = 2 2
(5I3 2
b= 0 a = c, b = 2c,
1 2 5 c 0
a 1
b = c 2 .
c 1
1
We may take as eigenvector 2 .
1
We find
1 1 .
P =
1
0 1
310 Appendix A
Since A = PDP1
A10 = PD10 P1
1
=
0
1 0
1 1
= 1 1023
.
1 1 0 1024 0 1 0 1024
getting
1 1 0
.
= 0 1
1
X 1
0 0 1
Thus
A =
XDX1
1 1 1 1 0 0 1 1 0
0 1 1 0 0 1
=
1
0 1
0 0 1 0 0 3 0 0 1
1 0 4
0 1 4 .
=
0 0 3
Bibliography
[Cul] CULLEN, C., Matrices and Linear Transformations, 2nd ed., New
York: Dover Publications, 1990.
[Lan] LANG, S., Introduction to Linear Algebra, 2nd ed., New York:
Springer-Verlag, 1986.
311
Index
312
INDEX 313
rank, 59
relation, 6
equivalence, 6
reflexive, 6
symmetric, 6
transitive, 6
residue classes, 14
right-handed, 120
scalar, 19
scalar multiplication
of a plane vector, 100
section formula, 102
sets, 1
Cartesian Product, 4
difference, 3
intersection, 3
subsets, 2
union, 2